Download as pdf or txt
Download as pdf or txt
You are on page 1of 132

Edventure

An initiative by IITians, for future IITians !!!

FOUNDATION BUILDER – IX
GEOMETRY

Head Office: 83, Heera Panna Mall, Hiranandani, Powai, Mumbai - 400 076
Geometry

Introduction

 Orthocenter is the point of intersection of the altitudes.

 Circumcentre is the point of intersection of the perpendicular bisectors of the sides.

 In centre is the point of intersection of the angular bisectors.

 Centroid is the point of intersection of the medians.

 The circumcentre of a triangle is equidistant from its vertices.

 The in centre of a triangle is equidistant from its sides.

 The centroid divides a median in the ratio 2 : 1.

 The orthocentre of a right angled triangle lies at the vertex containing the right angle.
A A

H O
C
B CB
Ortho centre Circumcentre OA = OB = OC
(i) (ii)
A
A
R Q
G
I
B P C
B Incentre C Centroid
AG : GP = 2 : 1
(iii) (iv)
Note:
 All four points are coincide for an equilateral triangle.
 Orthocentre (H), Centroid (G), Circumcentre (O) are always collinear points and G
divides OH in ratio 1 : 2.

Angle sum property for triangle


 Theorem: Prove that sum of all three angles is 180° or 2 right angles.

 Given: ABC

 To prove: A + B + C = 180°

FEAT Educational Ventures Pvt. Ltd.


H.O. 83, Heera Panna Mall, Near Jal-Vayu Vihar, Hiranandani Garden, Powai, Mumbai - 76 1
Call: +91-22-3226-8201 | SMS: +91-842-482-1664 | www.edventure.in | info@edventure.in
Foundation – IX Mathematics Geometry

 Construction: Draw PQ || BC, passes through point A.


P A Q
1 3
2

B C
1  B  alternate angle PQ || BC
 Proof: 
and 3  C      (I)
   PAQ is a line
   1 + 2 + 3 = 180° (linear pair application)
   B + 2 + C = 180°
   B + CAB + C = 180° = 2 right angles. Proved.

 Theorem: If one side of a triangle is produced then the exterior angle so formed is equal to
the sum of two interior opposite angles.
A
1

2 3 4
B C D
Means 4 = 1 + 2

 Proof: 3 = 180° – (1 + 2) ------ (I) (by angle sum property)
and BCD is a line
 3 + 4 = 180° (linear pair)
or 3 = 180° – 4 ------ (II)
by (I) & (II)
180° – (1 + 2) = 180° – 4
   1 + 2 = 4 Proved.

Congruent figures
 Two geometrical figures having exactly the same shape & size are known as congruent
figures. Lines, polygons, circles etc. can congruent.
Note:
 If radius of a circle is same as other circle then only both circles are congruent.
B
3
3 O
A
 Two line segment are congruent only when their length are equal.
A 4.0 cm B C 4.0 cm D

FEAT Educational Ventures Pvt. Ltd.


H.O. 83, Heera Panna Mall, Near Jal-Vayu Vihar, Hiranandani Garden, Powai, Mumbai - 76 2
Call: +91-22-3226-8201 | SMS: +91-842-482-1664 | www.edventure.in | info@edventure.in
Foundation – IX Mathematics Geometry

Congruent triangles
 Two triangles are congruent if and only if one of them can be made to superpose on the
other, so as to cover it exactly.

 Thus, congruent triangles are exactly identical.


For Example:
 If ABC ~  DEF then we have: A = D, B = E, C = F; and AB = DE, BC = EF
and AC = DE.
 If ABC ~  EDF then we have: A = E, B = D, C = F; and AB = ED, BC = DF
and AC = EF.
Note:
 Every triangle is congruent to itself, i.e., ABC ~ ABC.
 If ABC  DEF then DEF  ABC.
~ ~
 If ABC ~  DEF, and DEF ~  PQR, then ABC ~  PQR.
 'c.p.c.t.' for 'corresponding parts of congruent triangles'.

Criteria for congruent triangles


 SSS (Side Side Side)
A D
3 cm 4 cm
5 cm 3 cm

B 5 cm C F 4 cm E
  By SSS criteria ABC ~
 EDF
  A = E, B = D, C = F (c.p.c.t.)

 ASA (Angle Side Angle)


A A
y
6 cm

x y x
B 6 cm C D E
 By ASA criteria ABC ~
 DEF
 A = D, AB = DE, AC = DF (c.p.c.t.)

 AAS (Angle Angle Side)


D
A y
x 3 cm
y x
B 3 cm C F E
 By AAS, ABC ~ FDE
 C = E, AB = FD, AC = FE (c.p.c.t.)

FEAT Educational Ventures Pvt. Ltd.


H.O. 83, Heera Panna Mall, Near Jal-Vayu Vihar, Hiranandani Garden, Powai, Mumbai - 76 3
Call: +91-22-3226-8201 | SMS: +91-842-482-1664 | www.edventure.in | info@edventure.in
Foundation – IX Mathematics Geometry

 SAS (Side Angle Side)


A P
2 cm 2 cm 60° 3 cm
60°
B 3 cm C Q R
By SAS, ABC ~
 QPR
  A = Q, C = R, AC = QR (c.p.c.t.)

 RHS (Right Hypotenuse Side)


A 3 cm P
Q
3 cm 5 cm
5 cm

B C R
  By RHS, ABC ~ QPR
  A = Q, C = R, BC = PR (c.p.c.t.)
For Example:

R 4 cm
A P
35°
35° 4 cm 6 cm

B 6 cm C Q
Not congruent ( SSA is not a rule)

P A
cm

cm

70° 4.3 cm
3. 2

3. 2

70°
Q 4.3 cm R B C
SAS

D T R
40°
40° 7 cm 7 cm

E F X
AAS (not RHS)

A 2 cm D
C
2 cm
2 cm

30°
30°
B F 2 cm E
SAS (not RHS)

FEAT Educational Ventures Pvt. Ltd.


H.O. 83, Heera Panna Mall, Near Jal-Vayu Vihar, Hiranandani Garden, Powai, Mumbai - 76 4
Call: +91-22-3226-8201 | SMS: +91-842-482-1664 | www.edventure.in | info@edventure.in
Foundation – IX Mathematics Geometry

Theorem 1:
 If two angles and the included side of one triangle are equal to two angles and the included
side of other triangle, then both triangles are congruent.

 Given: ABC and DEF in which ABC = DEF, ACB = DFE and BC = EF.
D'
A D
D'

B C E F
 To prove: ABC ~
 DEF.

 Proof:
 Case-I
Let AC = DF.
In this case, AC = DF, BC = EF and C = F.
ABC ~  DEF (SAS-criteria)
 Case-II
If possible, let AC DF.
Then, construct D' F = AC. Join D' E.
Now, in ABC and D'EF, we have AC = D'F, BC = EF and C = F.
ABC ~  D'EF (SAS-criteria)
ABC = D'EF (c.p.c.t)
But, ABC = DEF (given)
D'EF = DEF.
This is possible only when D and D' coincide.
ABC ~  DEF.

Theorem 2:
 Two right-angled triangles are congruent if one side and the hypotenuse of the one are
respectively equal to the corresponding side and the hypotenuse of the other. (i.e. RHS)

 Given: Two right-angled triangles ABC & DEF in which B =E = 90°, BC = EF and AC = DF.

 To prove: ABC ~
 DEF.

 Construction: Produce DE to G such that GE = AB. Join GF.


A D

E F
B C

G
 Proof: In ABC and GEF, we have : AB = GE (construction),
BC = EF (given), B = FEG = 90°

FEAT Educational Ventures Pvt. Ltd.


H.O. 83, Heera Panna Mall, Near Jal-Vayu Vihar, Hiranandani Garden, Powai, Mumbai - 76 5
Call: +91-22-3226-8201 | SMS: +91-842-482-1664 | www.edventure.in | info@edventure.in
Foundation – IX Mathematics Geometry

   ABC 
~ GEF (SAS-criteria)
  A = G and AC = GF (c.p.c.t.)
Now, AC = GF and AC = DF GF = DF
   G = D A = D [ G = A]
Now, A = D, B = E 3rd C = 3rd F.
Thus, in ABC and DEF, we have:
BC = EF, AC = DF and C = F.
  ABC ~ DEF (SAS-criteria).

Ex.1 Prove that diagonal of a parallelogram divides it into two congruent triangles.
Sol. Let ABCD is a parallelogram and AC is a diagonal.
D C
2
3

4
1
A B
(By SSS): In ABC and ADC
AB = CD (opp. sides of ||gm)
BC = AD (opp. sides of ||gm)
AC = AC (common)
 By SSS, ABC ~  CDA proved
{other results : 1 = 2, 3 = 4, B = D (c.p.c.t.)}
(By ASA) : In ABC and ADC
1 = 2 (alternate)
AC = AC (common)
3 = 4 (alternate)
 By ASA, ABC ~  CDA {other results : B = D, AB = CD, BC = AD (c.p.c.t.)}
(By AAS) : In ABC and ADC
1 = 2 (alternate)
3 = 4 (alternate)
BC = AD (opp. sides)
 ABC ~  CDA {other results: AB = CD, B = D, AC = AC (c.p.c.t.)}
(By SAS): In ABC and ADC
AB = CD (opp. sides of ||gm)
1 = 2 (alternate)
AC = AC (common)
 ABC ~  CDA {other results: 3 = 4, BC = AD, B = D (c.p.c.t.)
We cannot use ‘RHS’ for this proof.
Note: ASS or SSA criteria for congruency are not valid.

FEAT Educational Ventures Pvt. Ltd.


H.O. 83, Heera Panna Mall, Near Jal-Vayu Vihar, Hiranandani Garden, Powai, Mumbai - 76 6
Call: +91-22-3226-8201 | SMS: +91-842-482-1664 | www.edventure.in | info@edventure.in
Foundation – IX Mathematics Geometry

Ex.2 In Fig. X and Y are two points on equal sides AB and AC of a ABC such that AX = AY.
Prove that XC = YB.
A

X Y

B C
Sol. In s AXC and AYB, we have
AX = AY [Given]
A = A [Common angle]
AC = AB [Given]
So, by SAS criterion of congruence
AXC ~ AYB
 XC = YB (c.p.c.t.)

Ex.3 In Fig. PQRS is a quadrilateral and T and U are respectively points on PS and RS such
PQ = RQ, PQT = RQU and TQS = UQS. Prove that QT = QU.
P
T

Q S

R
Sol. We have,
 PQT = RQU
and, TQS = UQS
 PQT + TQS = RQU + UQS
 PQS = RQS ------ (I)
Thus, in triangles PQS and RQS, we have
PQ = RQ [Given]
PQS = RQS [From (I)]
and, QS = QS [Common side]
Therefore, by SAS congruence criterion, we have
 PQS ~ RQS
 QPS = QRS (c.p.c.t.)
 QPT = QRU ------ (II)
Now, consider triangles QPT and QRS. In these two triangles, we have
QP = QR [Given]
 PQT = RQU [Given]
 QPT = QRU [From (II)]

FEAT Educational Ventures Pvt. Ltd.


H.O. 83, Heera Panna Mall, Near Jal-Vayu Vihar, Hiranandani Garden, Powai, Mumbai - 76 7
Call: +91-22-3226-8201 | SMS: +91-842-482-1664 | www.edventure.in | info@edventure.in
Foundation – IX Mathematics Geometry

Therefore, by ASA congruence criterion, we get


QPT ~ QRU
 QT = QU.

Ex.4 ABC is an isosceles triangle with AB = AC. Side BA is produced to D such that
AB = AD . Prove that BCD is a right angle.
Sol. Given: A ABC such that AB = AC. Side BA is produced to D such that AB = AD.
D

B C
Construction: Join CD.
To prove: BCD = 90º
Proof: In ABC, we have AB = AC
  Angles opp. to 
 ACB = ABC ------- (I) equal sides are equal 
 
Now, AB = AD [Given]
 AD = AC [ AB = AC]
Thus, in ADC, we have
AD = AC
 ACD = ADC ------- (II) [ Angles opp. to equal sides are equal]
Adding (I) and (II), we get
ACB + ACD = ABC + ADC
 BCD = ABC + BDC [ ADC = BDC, ABC = DBC]
Adding BCD 
 BCD + BCD = DBC + BCD + BDC  on both side 
 
 2 BCD = 180º [ Sum of the angles of a  is 180º]
Hence, BCD is a right angle.

Ex.5 In Fig. AC = BC, DCA = ECB and DBC = EAC.


D E

A C B
Prove that triangles DBC and EAC are congruent, and hence DC = EC.
Sol. We have,
DCA = ECB

FEAT Educational Ventures Pvt. Ltd.


H.O. 83, Heera Panna Mall, Near Jal-Vayu Vihar, Hiranandani Garden, Powai, Mumbai - 76 8
Call: +91-22-3226-8201 | SMS: +91-842-482-1664 | www.edventure.in | info@edventure.in
Foundation – IX Mathematics Geometry

 DCA + ECD = ECB + ECD


 ECA = DCB ------ (I)
Now, in s DBC and EAC, we have
DCB = ECA [From (I)]
BC = AC [Given]
and DBC = EAC [Given]
So, by ASA criterion of congruence
DBC ~ EAS
 DC = EC (c.p.c.t.)

Ex.6 If the altitudes from two vertices of a triangle to the opposite sides are equal, prove
that the triangle is isosceles.
Sol. Given: A ABC in which altitudes BE and CF from B and C respectively on AC and AB are
equal.
A

F E

B C
To prove: ABC is isosceles i.e. AB = AC
Proof: In s ABC and ACF, we have
AEB = AFC [Each equal to 90º]
BAE = CAF [Common angle]
and, BE = CF [Given]
So, by AAS criterion of congruence, we have
ABE ~ ACF
  Corresponding parts of 
 AB = AC congruent triangles are equal
 
Hence, ABC is isosceles.

Ex.7 In ABC, AB = AC and the bisectors of angles B and C intersect at point O. Prove that
BO = CO and the ray AO is the bisector of angle BAC.
Sol. In ABC, we have
AB = AC
A

B C
 Angles opposite to
 B = C  equal sides are equal 
 

FEAT Educational Ventures Pvt. Ltd.


H.O. 83, Heera Panna Mall, Near Jal-Vayu Vihar, Hiranandani Garden, Powai, Mumbai - 76 9
Call: +91-22-3226-8201 | SMS: +91-842-482-1664 | www.edventure.in | info@edventure.in
Foundation – IX Mathematics Geometry

1 1
 B = BC 
2 2
  OB and OC are bi sec tors of s B and C 
  OBC = OCB ------ (I)  1 1 
respectively  OBC  B & OCB  C
 2 2 
 OB = OC ------ (I) [ Sides opp. to equal s are equal]
Now, in ABO and ACO, we have
AB = AC [Given]
OBC = OCB [From (i)]
OB = OC [From (ii)]
So, by SAS criterion of congruence
ABO ~ ACO
 BAO = CAO [ Corresponding parts of congruent triangles are equal]
 AO is the bisector of BAC.

Ex.8 In Fig. BM and DN are both perpendiculars to the segments AC and BM = DN.
B A

N
R
M
C D
Prove that AC bisects BD.
Sol. In s BMR and DNR, we have BMR = DNR
[Each equal to 90º BM AC and DN AC]
BRM = DRN [Vert. opp. angles]
and, BM = DN [Given]
So, by AAS criterion of congruence
BMR ~ DNR
  Corresponding parts of 
 BR = DR congruent triangles are equal 
 
 R is the mid-point of BD.
Hence, AC bisects BD.

Ex.9 In Fig. BD and CE are two altitudes of a ABC such that BD = CE.
A

E D

B C
Prove that ABC is isosceles.
Sol. In ABD and ACE, we have

FEAT Educational Ventures Pvt. Ltd.


H.O. 83, Heera Panna Mall, Near Jal-Vayu Vihar, Hiranandani Garden, Powai, Mumbai - 76 10
Call: +91-22-3226-8201 | SMS: +91-842-482-1664 | www.edventure.in | info@edventure.in
Foundation – IX Mathematics Geometry

ADB = AEC = 90º [Given]


BAD = CAE [Common]
and, BD = CE [Given]
So, by AAS congruence criterion, we have
ABD ~ ACE
  Correspond ing parts of 
 AB = AC congruent triangles are equal 
 
Hence, ABC is isosceles.

Ex.10 If two isosceles triangles have a common base, the line joining their vertices bisects
them at right angles.
Sol. Given: Two isosceles triangles ABC and DBC having the common base BC such that
AB  AC and DB = DC.
To prove: AD (or AD produced) bisects BC at right angle.
A A

1 2 1 2
D

3 4 3 4
B C B C
E E

D
Proof: In s ABD and ACD, we have
AB = AC [Given]
BD = CD [Given]
AD = AD [Common side]
So, by SSS criterion of congruence
 ABD ~ ACD
  Corresponding parts of 
 1 = 2 ------ (I) congruent triangles are equal 
 
Now, in s ABE and ACE, we have
AB = AC [Given]
1 = 2 [From (i)]
and, AE = AE [Commoni side]
So, by SAS criterion of congruence,
ABE ~ ACE
  Corresponding parts of 
 BE = CE congruent triangles are equal 
 
and, 3 = 4
But, 3 + 4 = 180º [ Sum of the angles of a linear pair is 180º]
 2 3 = 180º [ 3 = 4]
 3 = 90º
 3 = 4 = 90º
Hence, AD bisects BC at right angles.

FEAT Educational Ventures Pvt. Ltd.


H.O. 83, Heera Panna Mall, Near Jal-Vayu Vihar, Hiranandani Garden, Powai, Mumbai - 76 11
Call: +91-22-3226-8201 | SMS: +91-842-482-1664 | www.edventure.in | info@edventure.in
Foundation – IX Mathematics Geometry

Ex.11 AD, BE and CF, the altitudes of ABC are equal. Prove that ABC is an equilateral
triangle
Sol. In right triangles BCE and BFC, we have
Hyp. BC = Hyp. BC
BE = CF [Given]
So, by RHS criterion of congruence,
A

F E

B D C
BCE ~ BFC.
  Correspond ing parts of 
 B = C congruent triangles are equal 
 
 AC = AB ------ (I) [ Sides opposite to equal angles are equal]
Similarly, ABD ~ ABE
 B =A [Corresponding parts of congruent triangles are equal]
 AC = BC ------ (II) [Sides opposite to equal angles are equal]
From (I) and (II), we get
AB = BC = AC
Hence, ABC is an equilateral triangle.

Ex.12 In Fig. AD = BC and BD = CA.


A B

D C
Prove that ADB = BCA and DAB = CBA.
Sol. In triangles ABD and ABC, we have
AD = BC [Given]
BD = CA [Given]
and AB = AB [Common]
So, by SSS congruence criterion, we have
  correspond ing parts of 
ABD ~ CBA  DAB = ABC congruent triangles are equal 
 
 DAB = CBA

FEAT Educational Ventures Pvt. Ltd.


H.O. 83, Heera Panna Mall, Near Jal-Vayu Vihar, Hiranandani Garden, Powai, Mumbai - 76 12
Call: +91-22-3226-8201 | SMS: +91-842-482-1664 | www.edventure.in | info@edventure.in
Foundation – IX Mathematics Geometry

Ex.13 Line-segment AB is parallel to another line-segment CD. O is the mid-point of AD (see


figure). Show that (I) AOB ~  DOC (II) O is also the mid-point of BC.
Sol. (i) Consider AOB and DOC
ABO = DCO (Alternate angles as AB || CD and BC is the transversal)
AOB = DOC (Vertically opposite angles)
OA = OD (Given)
Therefore, AOB ~ DOC (AAS rule)
C D

A B
(ii) OB = OC (c.p.c.t.)
So, O is the mid-point of BC.

Ex.14 In quadrilateral ABCD, AC = AD and AB bisects A. Show that ABC ~


 ABD. What
can you say about BC and BD?
C

1
A B
2

D
Sol. In ABC & ABD
AB = AB (common)
1 = 2 { AB is bisector of A}
AC = AD (Given)
By SAS, ABC ~
 ABD Proved
also BC = BD (c.p.c.t.)

Ex.15 AD and BC are equal perpendiculars to a line segment AB. Show that CD bisects AB.
B C

D A
Sol. To show CD bisect AB i.e. AO = OB
 in OAD and OBC
O = O (vertically opposite angles)
A = B = 90° (Given)

FEAT Educational Ventures Pvt. Ltd.


H.O. 83, Heera Panna Mall, Near Jal-Vayu Vihar, Hiranandani Garden, Powai, Mumbai - 76 13
Call: +91-22-3226-8201 | SMS: +91-842-482-1664 | www.edventure.in | info@edventure.in
Foundation – IX Mathematics Geometry

AD = BC (Given)
 By AAS, OAD ~  OBC
 OA = OB (c.p.c.t.)
 CD, bisects AB. Proved

Ex.16 Line l is the bisector of an angle A and B is any point on l. BP and BQ are
perpendiculars from B to the arms of A (see figure). Show that:

Q l
B
A P
(I) APB ~
 AQB
(II) BP = BQ or B is equidistant from the arms of A.
Sol. (i) In APB and AQB
P = Q = 90° (Given)
PAB = QAB (Given that ‘l’ bisect A)
AB = AB (Common)
 By AAS, APB  AQB. Proved
~
(ii) BP = BQ (c.p.c.t.) Proved.

Ex.17 In given figure, AC = AE, AB = AD and BAD = EAC. Show that BC = DE.
E
A
2
1

B D C
Sol. In ABC and ADE
AB = AD (Given)
  1  2 Given 
BAC = DAE  
1  DAC  2  DAC 
AC = AE (Given)
 By SAS, ABC ~  ADE
 BC = DE (c.p.c.t.) Proved.

FEAT Educational Ventures Pvt. Ltd.


H.O. 83, Heera Panna Mall, Near Jal-Vayu Vihar, Hiranandani Garden, Powai, Mumbai - 76 14
Call: +91-22-3226-8201 | SMS: +91-842-482-1664 | www.edventure.in | info@edventure.in
Foundation – IX Mathematics Geometry

Ex.18 In right triangle ABC, right angled at C, M is the mid-point of hypotenuse AB. C is
joined to M and produced to a point D such that DM = CM. Point D is joined to point B
(see figure). Show that:
D A

2 1
M

B C
(I) AMC  ~ BMD
(II) DBC is a right angle
(III) DBC ~  ACB
1
(IV) CM = AB
2
Sol. (i) In AMC and BMD
AM = MB (M is mid-point of AB)
1 = 2 (vertically opposite angles)
CM = MD (given)
 By SAS, AMC  MBD Proved.
~
(ii) ACM = MDB (c.p.c.t. of (i))
These are alternate angles
 DB || AC
So DBC + ACB = 180° (Co-interior angles)
 DBC + 90° = 180°
 DBC = 90° Proved.
A
D

(iii)

B C B C
In DBC & ACB
BC = BC (common)
DBC = ACB = 90°
DB = AC (c.p.c.t. of part (i))
 By SAS, DBC ~ ACB. Proved
(iv) DC = AB (c.p.c.t. of part (iii))
1
But CM = DC (given)
2
1
 CM = AB Proved.
2

FEAT Educational Ventures Pvt. Ltd.


H.O. 83, Heera Panna Mall, Near Jal-Vayu Vihar, Hiranandani Garden, Powai, Mumbai - 76 15
Call: +91-22-3226-8201 | SMS: +91-842-482-1664 | www.edventure.in | info@edventure.in
Foundation – IX Mathematics Geometry

Isosceles triangle
 A triangle in which two sides are equal & opposite angles of these two lines are also equal.
A

6 cm
6 cm

70° 70°
B C
AB = AC = 6 cm, B = C = 70°

1
Ex.19 Find BAC of an isosceles triangle in which AB = AC and B = of right angle.
3
1
Sol. B = C = (90) = 30°
3
 A + B + C = 180° (.p.)
A + 30° + 30° = 180° A = 120°.

Ex.20 In isosceles triangle DEF, DE = EF and E = 70° then find other two angles.
Sol.
D

E F
Let D = F = x
 D + E + F = 180° (angle sum property)
 x + 70° + x = 180°
 2x = 110°
 x = 55°
 D = F = 55°.
Theorem (2) : Angles opposite to equal sides of an isosceles triangle are equal.
A
12

B D C
Given: In ABC, AB = AC
To prove: B = C
Construction: Draw AD, bisector of A
 1 = 2
Proof: In ADB & ADC
AD = AD (Common)

FEAT Educational Ventures Pvt. Ltd.


H.O. 83, Heera Panna Mall, Near Jal-Vayu Vihar, Hiranandani Garden, Powai, Mumbai - 76 16
Call: +91-22-3226-8201 | SMS: +91-842-482-1664 | www.edventure.in | info@edventure.in
Foundation – IX Mathematics Geometry

1 = 2 (by construction)
AB = AC
By SAS, ADB ~  ADC
 B = C (c.p.c.t.) Proved.
Note:
Other result : ADB = ADC (c.p.c.t.)
But ADB + ADC = 180° (linear pair)
 ADB = ADC = 90° AD BC
and BD = DC (c.p.c.t.) AD is median
 we can say AD is perpendicular bisector of BC or we can say in isosceles , median
is angle bisector and perpendicular to base also.

Ex.21 ABC and DBC are two isosceles triangles on the same base BC and vertices A and
D are on the same side of BC (see fig.). If AD is extended to intersect BC at P. Show
that
A

B P C
(I) ABD ~  ACD
(II) ABP ~  ACP
(III) AP bisects A as well as D
(IV) AP is the perpendicular bisector of BC.
Sol. (i) In ABD & ACD
AB = AC (ABC is isosceles 
AD = AD (Common)
BD = DC (DBC is isosceles )
By SSS, ABD  ACD Proved.
~
(ii) In ABP & ACP
AB = AC (ABC is isosceles 
ABP = ACP {ABC is isosceles }
AP = AP (common)
By SAS, ABP ~
 ACP Proved.
(iii)  BAP = CAP (c.p.c.t. of part (ii))
A is bisected by AP
and ADB = ADC (c.p.c.t. of part (ii))
CD is bisected by AP.
(iv) APB = APC (c.p.c.t. of part (ii))
but APB + APC = 180° (linear pair)
APB + APB = 180°
2APB = 180°

FEAT Educational Ventures Pvt. Ltd.


H.O. 83, Heera Panna Mall, Near Jal-Vayu Vihar, Hiranandani Garden, Powai, Mumbai - 76 17
Call: +91-22-3226-8201 | SMS: +91-842-482-1664 | www.edventure.in | info@edventure.in
Foundation – IX Mathematics Geometry

APB = 90° = APC


also PB = PC (c.p.c.t. of part (ii))
AP is perpendicular bisector of BC. Proved.

Ex.22 Two sides AB and BC and median AM of one triangle ABC are respectively equal to
sides PQ and QR and median PN of PQR (see figure). Show that:
A P

B M C Q N R
(I) ABM ~  PQN
(II) ABC ~  PQR
Sol. (i) In ABM & PQN
AB = PQ (given)
AM = PN (given)
BC QR
BM = QN ( BC = QR   )
2 2
By SSS, ABM ~  PQN Proved.
(ii) In ABC & PQR
AB = PQ (given)
B = Q (c.p.c.t. of part (i))
BC = QR (given)
By SAS, ABC ~  PQR Proved.

Some more results based on congruent triangles


 If two sides of a triangle are unequal, then the longer side has the greater angle opposite to
it.

 In a triangle, the greater angle has the longer side opposite to it.

 Of all the line segments that can be drawn to a given line, from a point not lying on it, the
perpendicular line segment is the shortest.

 The sum of any two sides of a triangle is greater than its third side.

 The difference between any two sides of a triangle is less than its third side.

 Exterior angle is greater than one opposite interior angle.

FEAT Educational Ventures Pvt. Ltd.


H.O. 83, Heera Panna Mall, Near Jal-Vayu Vihar, Hiranandani Garden, Powai, Mumbai - 76 18
Call: +91-22-3226-8201 | SMS: +91-842-482-1664 | www.edventure.in | info@edventure.in
Foundation – IX Mathematics Geometry

Ex.23 Find the relation between angles in figure.


x
7 cm
5 cm

y 8 cm z
Sol.  yz > xz > xy
x > y > z. ( Angle opposite to longer side is greater)

Ex.24 Find the relation between the sides of triangle in figure.


D
80°

70° 30° F
E
Sol.  D > E > F
EF > DF > DE { side opposite to greater angle is longer}

Ex.25 Find ACD then what the relation is between


A

70° 40°
B C D
(I) ACD, ABC
(II) ACD & A
Sol. ACD + 40° = 180° (linear pair)
ACD = 140° Ans.
also A + B = ACD (exterior angle = sum of opp. interior angles)
A + 70° = 140° A = 140° – 70°
A = 70°
Now ACD > B Ans.
ACD > A Ans.

Ex.26 In Fig. E > A and C > D.


D
E
B

C
A
Prove that AD > EC.

FEAT Educational Ventures Pvt. Ltd.


H.O. 83, Heera Panna Mall, Near Jal-Vayu Vihar, Hiranandani Garden, Powai, Mumbai - 76 19
Call: +91-22-3226-8201 | SMS: +91-842-482-1664 | www.edventure.in | info@edventure.in
Foundation – IX Mathematics Geometry

Sol. In ABE, it is given that


E > A
 AB > BE ------ (I)
In BCD, it is given that
C > D
 BD > BC ------ (II)
Adding (i) and (ii), we get
AB + BD > BE + BC AD > EC

Ex.27 AB and CD are respectively the smallest and longest sides of a quadrilateral ABCD
(see figure). Show that A > C.
D

B C
Sol. Draw diagonal AC.
D

A 2
1

4
3
B C
In ABC, AB < BC { AB is smallest}
 3 < 1 ------ (I) {angle opp. to longer side is larger}
Also in ADC
AD < CD  CD is longest
 4 < 2 ------ (II)
adding equation (I) & (II)
3 + 4 < 1 + 2
C < A
or A > C Proved.

FEAT Educational Ventures Pvt. Ltd.


H.O. 83, Heera Panna Mall, Near Jal-Vayu Vihar, Hiranandani Garden, Powai, Mumbai - 76 20
Call: +91-22-3226-8201 | SMS: +91-842-482-1664 | www.edventure.in | info@edventure.in
Foundation – IX Mathematics Geometry

Ex.28 In given figure, PR > PQ and PS bisects QPR. Prove that PSR > PSQ.
P

Q S R
Sol. In PQR, PR > PQ
 Q > R ------ (I)
{angle opposite to longer side is greater}
and 1 = 2 ( PS is bisector) ------ (II)
P

12

Q S R
Now for PQS, PSR = Q + 1 ------ (III)
{exterior angle = sum of opposite interior angle}
P
1

Q S
& for PSR, PSQ = R + 2 ------ (IV)
By equation (I), (II), (III), (IV), PSR > PSQ Proved.

Ex.29 AD, BE and CF, the altitudes of ABC are equal. Prove that ABC is an equilateral
triangle
Sol. In right triangles BCE and BFC, we have
Hyp. BC = Hyp. BC
BE = CF [Given]
So, by RHS criterion of congruence,
A

F E

B D C
BCE ~ BFC.
  Corresponding parts of 
 B = C congruent triangles are equal 
 

FEAT Educational Ventures Pvt. Ltd.


H.O. 83, Heera Panna Mall, Near Jal-Vayu Vihar, Hiranandani Garden, Powai, Mumbai - 76 21
Call: +91-22-3226-8201 | SMS: +91-842-482-1664 | www.edventure.in | info@edventure.in
Foundation – IX Mathematics Geometry

 AC = AB ------ (I) [ Sides opposite to equal angles are equal]


Similarly, ABD ~ ABE
 B =A [Corresponding parts of congruent triangles are equal]
 AC = BC ------ (II) [Sides opposite to equal angles are equal]
From (i) and (ii), we get
AB = BC = AC
Hence, ABC is an equilateral triangle.

Ex.30 In Fig. AD = BC and BD = CA.


A B

D C
Prove that ADB = BCA and DAB = CBA.
Sol. In triangles ABD and ABC, we have
AD = BC [Given]
BD = CA [Given]
and AB = AB [Common]
So, by SSS congruence criterion, we have
  corresponding parts of 
ABD ~ CBA  DAB = ABC congruent triangles are equal 
 
 DAB = CBA

Ex.31 In Fig. PQ > PR. QS and RS are the bisectors of Q and R respectively.
P

Q R
Prove that SQ > SR.
Sol. In PQR, we have
PQ > PR [Given]
Angle opp. to larger side 
 PRQ > PQR  of a triangle is greater 
 
1 1
 PRQ > PQR
2 2
 RS and QS are bi sec tors of 
 SRQ > SQR  PRQ are PQR respectively 
 
 SQ > SR [ Side opp. to greater angle is larger]

FEAT Educational Ventures Pvt. Ltd.


H.O. 83, Heera Panna Mall, Near Jal-Vayu Vihar, Hiranandani Garden, Powai, Mumbai - 76 22
Call: +91-22-3226-8201 | SMS: +91-842-482-1664 | www.edventure.in | info@edventure.in
Foundation – IX Mathematics Geometry

Ex.32 In Fig.
L

M N
x° y°
If x > y, show that M > N.
Sol. We have,
LMN + xº = 180º ------ (I) [Angles of a linear pair]
 LNM + yº = 180º ------ (II) [Angles of a linear pair]
 LMN + xº = LNM + yº
But x > y. Therefore,
LMN < LNM
 LNM > LMN
 LM > LN [ Side opp. to greater angle is larger]

Ex.33 In Fig. AB > AC. Show that AB > AD.

B D C
Sol. In ABC, we have
AB > AC [Given]
 ACB > ABC ------ (I) [ Angle opp. to larger side is greater]
Now, in ACD, CD is produced to B, forming an ext ADB.
 Exterior angle of Δ is greater 
 ADB > ACD  than each of interior opp. angle 
 
 ADB > ACB ------ (II) [ ACD = ACB]
From (i) and (ii), we get
ADB > ABC
 ADB > ABD [ ABC = ABD]
 AB > AD [ Side opp. to greater angle is larger]

FEAT Educational Ventures Pvt. Ltd.


H.O. 83, Heera Panna Mall, Near Jal-Vayu Vihar, Hiranandani Garden, Powai, Mumbai - 76 23
Call: +91-22-3226-8201 | SMS: +91-842-482-1664 | www.edventure.in | info@edventure.in
Foundation – IX Mathematics Geometry

Ex.34 Prove that any two sides of a triangle are together greater than twice the median
drawn to the third side.
Sol. Given: A  ABC in which AD is a median.
A

D
B C

E
To prove: AB + AC > 2 AD
Construction: Produce AD to E such that AD = DE. Join EC.
Proof: In s ADB and EDC, we have
AD = DE [By construction]
BD = DC [ D is the mid point of BC]
and, ADB = EDC [Vertically opp. angles]
So, by SAS criterion of congruence
ADB ~ EDC
Corresponding parts of 
AB = EC congruenttriangles are equal
 
Now in AEC, we have
AC + EC > AE [ Sum of any two sides of a  is greater than the third]
 AC + AB > 2 AD  AD  DE  AE  AD  DE  2 AD and EC  AB

Ex.35 In Fig. PQR is a triangle and S is any point in its interior,


show that SQ + SR < PQ + PR.
P

T
S

Q R
Sol. Given: S is any point in the interior of PQR.
To Prove: SQ + SR < PQ + PR
Construction: Produce QS to meet PR in T.
Proof: In PQT, we have
 Sum of the two sides of a Δ
PQ + PT > QT is greater than the third side 
 
 PQ + PT > QS + ST ------ (I) [ QT = QS + ST]
In RST, we have
ST + TR > SR ------ (II)
Adding (i) and (ii), we get
PQ + PT + ST + TR > SQ + ST + SR

FEAT Educational Ventures Pvt. Ltd.


H.O. 83, Heera Panna Mall, Near Jal-Vayu Vihar, Hiranandani Garden, Powai, Mumbai - 76 24
Call: +91-22-3226-8201 | SMS: +91-842-482-1664 | www.edventure.in | info@edventure.in
Foundation – IX Mathematics Geometry

 PQ + (PT + TR) > SQ + SR


 PQ + PR > SQ + SR SQ + SR < PQ + PR.

Ex.36 In PQR S is any point on the side QR. Show that PQ + QR + RP > 2 PS.
P

Q S R
Sol. In PQS, we have
PQ + QS > PS ------ (I)
[ Sum of the two sides of a  is greater than the third side]
Similarly, in PRS, we have
RP + RS > PS ------ (II)
Adding (i) and (ii), we get
(PQ + QS) + (RP + RS) > PS + PS
 PQ + (QS + RS) + RP > 2 PS
 PQ + QR + RP > 2 PS [ QS + RS = QR]

Ex.37 In Fig. T is a point on side QR of PQR and S is a point such that RT = ST.
Q

T
S

R P
Prove that PQ + PR > QS.
Sol. In PQR, we have
PQ + PR > QR
 PQ + PR > QT + RT [ QR = QT + RT]
 PQ + PR > QT + ST ------ (I) [ RT = ST (Given)]
In QST, we have
QT + ST > QS ------ (II)
From (I) and (II), we get
PQ + PR > QS.

FEAT Educational Ventures Pvt. Ltd.


H.O. 83, Heera Panna Mall, Near Jal-Vayu Vihar, Hiranandani Garden, Powai, Mumbai - 76 25
Call: +91-22-3226-8201 | SMS: +91-842-482-1664 | www.edventure.in | info@edventure.in
Foundation – IX Mathematics Geometry

Ex.38 Find OBA in given figure


Sol.

198°
O
A B

 AOB + 198° = 360°


AOB = 360° – 198° = 162°
and OA = OB = radius of circle
A = B = x (let)
 x + x + 162° = 180° (a.s.p.)
2x + 18°
x = 9°
 OBA = 9°.

Important points to be remembered


 A plane figure bounded by three lines in a plane is called a triangle.

 A triangle, no two of whose sides are equal is called a scalene triangle.

 A triangle whose two sides are equal is called an isosceles triangle.

 A triangle whose sides are equal is also called an equilateral triangle.

 A triangle with one angle a right angle is called a right angled triangle.

 The sum of the three angles of a triangle is 180º.

 If a side of a triangle is produced, the exterior angle so formed is equal to the sum of the two
interior opposite angles.

 If two triangles ABC and DEF are congruent under the correspondence A D, B  E and
C  F, then we write  ABC ~ DEF or ABC DEF.

 Two triangles are congruent if two sides and the included angle of one are equal to the
corresponding sides and the included angle of the other triangle (SAS congruence criterion).

 Two triangles are congruent if two angles and the included side of one triangle are equal to
the corresponding two angles and the included side of the other triangle (ASA congruence
criterion).

 If any two angles and non-included side of one triangle are equal to the corresponding
angles and side of another triangle, then the triangles are congruent (AAS congruence
criterion).

FEAT Educational Ventures Pvt. Ltd.


H.O. 83, Heera Panna Mall, Near Jal-Vayu Vihar, Hiranandani Garden, Powai, Mumbai - 76 26
Call: +91-22-3226-8201 | SMS: +91-842-482-1664 | www.edventure.in | info@edventure.in
Foundation – IX Mathematics Geometry

 If three sides of one triangle are equal to three of the other triangle, then the two triangles
are congruent (SSS congruence criterion).

 If in two right triangles, hypotenuse and one side of a triangle are equal to the hypotenuse
and one side of other triangle, then the two triangles are congruent (RHS congruence
criterion).

 Angles opposite to equal sides of a triangle are equal.

 If the altitude from one vertex of a triangle bisects the opposite sides, then the triangle is
isosceles.

 In an isosceles triangle altitude from the vertex bisects the base.

 If the bisector of the vertical angle of a triangle bisects the opposite side, then the triangle is
isosceles.

 If the altitudes of a triangle are equal, then it is equilateral.

 In a triangle, side opposite to the larger angle is longer.

 Sum of any two sides of a triangle is greater than the third side.

Concept of Similarity
 Geometric figures having the same shape but different sizes are known as similar figures.
Two congruent figures are always similar but similar figures need not be congruent.

Illustration 1:
 Any two line segments are always similar but they need not be congruent. They are
congruent, if their lengths are equal.

Illustration 2:
 Any two circles are similar but not necessarily congruent. They are congruent if there are
equal.

Illustration 3:
 Any two square are similar (see fig. (i))

Fig. (i) Fig. (ii)

 Any two equilateral triangles are similar (see fig. (ii))

FEAT Educational Ventures Pvt. Ltd.


H.O. 83, Heera Panna Mall, Near Jal-Vayu Vihar, Hiranandani Garden, Powai, Mumbai - 76 27
Call: +91-22-3226-8201 | SMS: +91-842-482-1664 | www.edventure.in | info@edventure.in
Foundation – IX Mathematics Geometry

Similar polygons

Definition:
 Two polygons are said to be similar to each other, if
 their corresponding angles are equal, and
 the lengths of their corresponding sides are proportional.

 If two polygons ABCDE and PQRST are similar, then from the above definition it follows that:
Angle at A = Angle at P, Angles at B = Angle at Q, Angle at C = Angle at R,
Angle at D = Angle at S, Angle at E = Angle at T
AB BC CD DE
and,   
PQ QR RS ST
If two polygons ABCDE and PQRST, are similar, we write ABCDE PQRST .
Here, the symbol ' ' stands for is similar to.

Similar triangles and their properties

Definition:
 Two triangles are said to be similar, if their
 Corresponding angles are equal and,
 Corresponding sides are proportional.

 Two triangles ABC and DEF are similar, if


 A  D , B  E , C  F and,
AB BC AC
  
DE EF DF

Some basic results on proportionality

Basic proportionality theorem or Thales theorem:


 If a line is drawn parallel to one side of a triangle intersecting the other two sides, then it
divides the two sides in the same ratio.

 Given: A triangle ABC in which DE || BC , and intersects AB in D and AC in E.

AD AE
 To Prove: 
DB EC

FEAT Educational Ventures Pvt. Ltd.


H.O. 83, Heera Panna Mall, Near Jal-Vayu Vihar, Hiranandani Garden, Powai, Mumbai - 76 28
Call: +91-22-3226-8201 | SMS: +91-842-482-1664 | www.edventure.in | info@edventure.in
Foundation – IX Mathematics Geometry

 Construction: Join BE, CD and draw EF  BA and DG  CA .

 Proof:
Since EF is perpendicular to AB. Therefore, EF is the height of triangles ADE and DBE.
1 1
Now,  ADE   base  height    AD  EF 
2 2
1 1
and,  DBE   base  height   DB  EF 
2 2
1
Area  ADE  2  AD  EF  AD
    ----- (I)
Area  DBE  1
DB  EF  DB
2
Similarly, we have
1
Area  ADE  2  AE  DG  AE
  ----- (II)
Area  DEC  1
 EC  DG  EC
2
But, DBE and DEC are on the same base DE and between the same parallels DE and
BC.
  Area (DBE) = Area (DEC)
1 1
   [Taking reciprocals of both sides]
Area  DBE  Area  DEC 

Area  ADE  Area  ADE 


   [Multiplying both sides by Area (ADE)]
Area  DBE  Area  DEC 
AD AE
   [Using (I) and (II)]
DB EC

 Corollary: If in a ABC, a line DE || BC , intersects AB in D and AC in E, then:


AB AC
 
AD AE
AB AC
 
DB EC

 Proof:
 From the basic proportionality theorem, we have
AD AE

DB EC
DB EC
  [Taking reciprocals of both sides]
AD AE

FEAT Educational Ventures Pvt. Ltd.


H.O. 83, Heera Panna Mall, Near Jal-Vayu Vihar, Hiranandani Garden, Powai, Mumbai - 76 29
Call: +91-22-3226-8201 | SMS: +91-842-482-1664 | www.edventure.in | info@edventure.in
Foundation – IX Mathematics Geometry

DB EC
 1  1 [Adding 1 on both sides]
AD AE
AD  DB AE  EC
 
AD AE
AB AC
 
AD AE
 From the basic proportionality theorem, we have
AD DE

DB EC
AD AE
 1  1 [Adding 1 on both sides]
DB EC
AD  DB AE  EC
 
DB EC
AB AC
 
DB EC
So, If in a ABC, DE || BC and intersect AB in D and AC in E, then we have
AD AE
 
DB EC
DB EC
 
AD AE
AB AC
 
AD AE
AD AE
 
AB AC
AB AC
 
DB EC
DB EC
 
AB AC

Converse of basic proportionality theorem:


 If a line divides any two sides of a triangle in the same ratio, then the line must be parallel to
the third side.

AD AE
 Given: A ABC and a line l intersecting AB in D and AC in E, such that 
DB EC

 To prove: || BC i.e. DE || BC

 Proof:

FEAT Educational Ventures Pvt. Ltd.


H.O. 83, Heera Panna Mall, Near Jal-Vayu Vihar, Hiranandani Garden, Powai, Mumbai - 76 30
Call: +91-22-3226-8201 | SMS: +91-842-482-1664 | www.edventure.in | info@edventure.in
Foundation – IX Mathematics Geometry

If possible, let DE be not parallel to BC. Then, there must be another line parallel to BC. Let
DF || BC
Since DF || BC . Therefore from Basic Proportionality Theorem, we get
AD AF
 ----- (I)
DB FC
AD AE
But,  (Given) ----- (II)
DB EC
From (I) and (II), we get
AF AE

FC EC
AF AE
 1 1 [Adding 1 on both sides]
FC EC
AF  FC AE  EC
 
FC EC
AC AC
 
FC EC
 FC  EC
This is possible only when F and E coincide i.e. DF is the line l itself.
But, DF || BC . Hence, 1|| BC .

KP 4
Ex.39 In fig., PQ is parallel to MN. If  and KN = 20.4 cm. Find KQ.
PM 11

Sol. In KMN, we have


PQ || MN
KP KQ
  [By Thale’s Theorem]
PM QN
KP KQ
 
PM KN  KQ
4 KQ
 
13 20.4  KQ
 4  20.4  KQ   13KQ
 81.6 – 4KQ = 13 KQ
 17KQ = 81.6
81.6
 KQ   4.8 cm
17

FEAT Educational Ventures Pvt. Ltd.


H.O. 83, Heera Panna Mall, Near Jal-Vayu Vihar, Hiranandani Garden, Powai, Mumbai - 76 31
Call: +91-22-3226-8201 | SMS: +91-842-482-1664 | www.edventure.in | info@edventure.in
Foundation – IX Mathematics Geometry

AD 3
Ex.40 In a given ABC, DE || BC and  . If AC = 5.6, find AE.
DB 5
Sol. In ABC, we have
DE || BC
AD AE
  [By Thale’s Theorem]
DB EC

AD AE
 
DB AC  AE
3 AE
  [ AC = 5.6]
5 5.6  AE
 3(5.6 – AE) = 5AE
 16.8 – 3AE = 5AE
 8AE = 16.8
16.8
 AE  cm  21 cm
8

Ex.41 In fig., DE || BC. If AD = x, DB = x – 2, AE = x + 2 and EC = x – 1, find the value of x.

Sol. In ABC, we have


DE || BC
AD AE
  [By Thale’s Theorem]
DB EC
x x2
 
x  2 x 1
 x  x  1   x  2  x  2 

 x2  x  x2  4
 x4

FEAT Educational Ventures Pvt. Ltd.


H.O. 83, Heera Panna Mall, Near Jal-Vayu Vihar, Hiranandani Garden, Powai, Mumbai - 76 32
Call: +91-22-3226-8201 | SMS: +91-842-482-1664 | www.edventure.in | info@edventure.in
Foundation – IX Mathematics Geometry

Ex.42 In fig., LM || AB. If AL = x – 3, AC = 2x, BM = x – 2 and BC = 2x + 3, find the value of x.

Sol. In ABC, we have


LM || AB
AL MB
  [By Thale’s Theorem]
LC MC
AL BM
  
AC  AL BC  BM
x3 x2
  
2x   x  3   2x  3    x  2 
x 3 x 2
 
x3 x5
  x  3  x  5    x  2  x  3 
 x2  2x  15  x2  x  6
 x9

AE BF
Ex.43 In fig., if EF || DC || AB. Prove that  .
ED FC
Sol. Given: EF || DC || AB in the given figure.
AE BF
To Prove: 
ED FC
Construction: Produce DA and CB to meet at P (say).

Proof: In PEF, we have


AB || EF
PA PB
  [By Thale’s Theorem]
AE BF
PA PB
 1  1 [Adding 1 on both sides]
AE BF
PA  AE PB  BF
 
AE BF

FEAT Educational Ventures Pvt. Ltd.


H.O. 83, Heera Panna Mall, Near Jal-Vayu Vihar, Hiranandani Garden, Powai, Mumbai - 76 33
Call: +91-22-3226-8201 | SMS: +91-842-482-1664 | www.edventure.in | info@edventure.in
Foundation – IX Mathematics Geometry

PE PF
  ------ (I)
AE BF
In PDC, we have
EF || DC
PE PF
  ------ (II) [By Basic Proportionality Theorem]
ED FC
On dividing equation (I) by equation (II), we get
PE PF
AE  BF
PE PF
ED FC
ED FC
 
AE BF
AE BF
 
ED FC

Ex.44 Let X is any point on the side BC of a triangle ABC. If XM, XN are drawn parallel to BA
and CA meeting CA, BA in M, N respectively; MN meets BC produced in T, prove that
TX2  TB  TC .
Sol. In TXM, we have

XM || BN
TB TM
  ----- (I)
TX TN
In TMC, we have
XN || CM
TX TN
  ----- (II)
TC TM
From equations (I) and (II), we get
TB TX

TX TC
 TX2  TX  TC

Ex.45 ABCD is a parallelogram, P is a point on side BC and DP when produced meets AB


produced at L prove that
DP DC DL AL
(I)  (II) 
PL BL DP DC
Sol. Given:

FEAT Educational Ventures Pvt. Ltd.


H.O. 83, Heera Panna Mall, Near Jal-Vayu Vihar, Hiranandani Garden, Powai, Mumbai - 76 34
Call: +91-22-3226-8201 | SMS: +91-842-482-1664 | www.edventure.in | info@edventure.in
Foundation – IX Mathematics Geometry

A parallelogram ABCD in which P is a point on side BC such that DP produced meet AB


produced at L.

To Prove:
DP DC DL AL
(i)  (ii) 
PL BL DP DC
Proof:
(i) In ALD, we have
BP || AD
LB LP
 
BA PD
BL PL
 
AB DP
BL PL
  [ AB = DC]
DC DP
DP DC
  [Taking reciprocals of both sides]
PL BL
(ii) From (i), we have
DP DC

PL BL
PL BL
  [Taking reciprocals of both sides]
DP DC
PL BL
  [ DC = AB]
DP AB
PL BL
 1 1 [Adding 1 on both sides]
DP AB
DP  PL BL  AB
 
DP AB
DL AL
 
DP AB
DL AL
 
DP DC

AE BF
Ex.46 In fig., EF || AB || DC. Prove that  .
ED FC

Sol. We have,

FEAT Educational Ventures Pvt. Ltd.


H.O. 83, Heera Panna Mall, Near Jal-Vayu Vihar, Hiranandani Garden, Powai, Mumbai - 76 35
Call: +91-22-3226-8201 | SMS: +91-842-482-1664 | www.edventure.in | info@edventure.in
Foundation – IX Mathematics Geometry

EF || AB || DC
 EP || DC
Thus, in ADC, we have
EP || DC
Therefore, by basic proportionality theorem, we have
AE AP
 ------ (I)
ED PC
Again, EF || AB || DC
 FP || AB
Thus, in CAB, we have
FP || BA
Therefore, by basic proportionality theorem, we have
BF AP
 ------ (II)
FC PC
From (i) and (ii), we have
AE BF

ED FC

AR AQ
Ex.47 In fig., if PQ || BC and PR || CD. Prove that  .
AD AB

Sol. In ABC, we have


PR || CD [Given]
Therefore, by basic proportionality theorem, we have
AQ AP
 ------ (I)
AB AC
In ACD, we have
PR || CD
Therefore, by basic proportionality theorem, we have
AP AR
 ------ (II)
AB AD
From (i) and (ii), we obtain that
AQ AR AR AQ
 or 
AB AD AD AB

FEAT Educational Ventures Pvt. Ltd.


H.O. 83, Heera Panna Mall, Near Jal-Vayu Vihar, Hiranandani Garden, Powai, Mumbai - 76 36
Call: +91-22-3226-8201 | SMS: +91-842-482-1664 | www.edventure.in | info@edventure.in
Foundation – IX Mathematics Geometry

BE BC
Ex.48 In fig., DE || AC and DC || AP. Prove that  .
ED CP
Sol. In BPA, we have

DC || AP [Given]
Therefore, by basic proportionality theorem, we have
BC BD
 ------ (I)
CP DA
In BCA, we have
DE || AC [Given]
Therefore, by basic proportionality theorem, we have
BE BD
 ------ (II)
EC DA
From (I) and (II), we get
BC BE BE BC
 or 
CP EC EC CP

Ex.49 In fig. DE || BC and CD || EF. Prove that AD2  AB  AF .


Sol. In ABC, we have

DE || BC
AB AC
  ------ (I)
AD AE
In ADC, we have
FE || DC
AD AC
  ------ (II)
AF AE
From (i) and (ii), we get
AB AD

AD AF
 AD2  AB  AF

FEAT Educational Ventures Pvt. Ltd.


H.O. 83, Heera Panna Mall, Near Jal-Vayu Vihar, Hiranandani Garden, Powai, Mumbai - 76 37
Call: +91-22-3226-8201 | SMS: +91-842-482-1664 | www.edventure.in | info@edventure.in
Foundation – IX Mathematics Geometry

Ex.50 D and E are respectively the points on the sides AB and AC of a ABC such that
AB  5.6 cm. AD  1.4 cm, AC  7.2 cm and AE  1.8 cm, show that DE || BC.
Sol. We have,

AB = 5.6 cm, AD = 1.4 cm, AC = 7.2 cm and AE = 1.8 cm.


 BD = AB – AD = (5.6 – 1.4) cm 4.2 cm
and, EC = AC – AE = (7.2 – 1.8) cm = 5.4 cm
AD 1.4 1 AE 1.8 1
Now,   and  
DB 4.2 3 EC 5.4 3
AD AE
 
DB EC
Thus, DE divides sides AB and AC of ABC in the same ratio. Therefore, by the converse of
Basic Proportionality Theorem, we have DE || BC

Ex.51 Any point X inside DEF is joined to its vertices. From a point P in DX, PQ is drawn
parallel to DE meeting XE at Q and QR is drawn parallel to EF meeting XF in R. Prove
that PR || DF.
Sol. A DEF and a point X inside it. Point X is joined to the vertices D, E and F. P is any point on
DX. PQ || DE and QR || EF.
To Prove: PR || DF
Construction: Join PR.
Proof: In XED, we have

PQ || DE
XP XQ
  ------ (I) [By Thale’s Theorem]
PD QE
In XEF, we have
QR || EF
XQ XR
  ------ (II) [By Thale’s Theorem]
QE RF
From (i) and (ii), we have

FEAT Educational Ventures Pvt. Ltd.


H.O. 83, Heera Panna Mall, Near Jal-Vayu Vihar, Hiranandani Garden, Powai, Mumbai - 76 38
Call: +91-22-3226-8201 | SMS: +91-842-482-1664 | www.edventure.in | info@edventure.in
Foundation – IX Mathematics Geometry

XP XR

PD RF
Thus, in XFD, points R and P are dividing sides XF and XD in the same ratio. Therefore, by
the converse of Basic Proportionality Theorem, we have PR || DF

Ex.52 Two triangles ABC and DBC lie on the same side of the base BC. From a point P on
BC, PQ || AB and PR || BD are drawn. They meet AC in Q and DC in R respectively.
Prove that QR || AD.
Sol. Given: Two triangles ABC and DBC lie on the same side of the base BC. Points P, Q and R
are points on BC, AC and CD respectively such that PR || BD and PQ || AB.

To Prove: QR || AD
Proof: In ABC, we have
PQ || AB
CP CQ
  ----- (I) [By Basic Proportionality Theorem]
PB QA
In BCD, we have
PR || BD
CP CR
  ----- (II) [By Thale’s Theorem]
PB RD
From (I) and (II), we have
CQ CR

QA RD
Thus, in ACD, Q and R are points on AC and CD respectively such that
CQ CR

QA RD
 QR || AD [By the converse of Basic proportionality theorem]

Ex.53 Let ABC be a triangle and D and E be two points on side AB such that AD = BE.
If DP || BC and EQ || AC, then prove that PQ || AB.
Sol. In ABC, we have

DP || BC and EQ || AC

FEAT Educational Ventures Pvt. Ltd.


H.O. 83, Heera Panna Mall, Near Jal-Vayu Vihar, Hiranandani Garden, Powai, Mumbai - 76 39
Call: +91-22-3226-8201 | SMS: +91-842-482-1664 | www.edventure.in | info@edventure.in
Foundation – IX Mathematics Geometry

AD AP BE BQ
  and 
DB PC EA QC
AD AP AD BQ
  and  EA  ED  DA  ED  BE  BD  AD  BE
DB PC DB QC 
AP BQ
 
PC QC
 In a ABC, P and Q divide sides CA and CB respectively in the same ratio.
 PQ || AB.

Ex.54 In figure, ABC is a triangle in which AB = AC. Points D and E are points on the sides
AB and AC respectively such that AD = AE. Show that the points B, C, E and D are
concyclic.
Sol. In order to prove that the points B, C, E and D are concyclic, it is sufficient to show that 
 ABC + CED = 180º and ACB + BDE = 180º.
In ABC, we have
AB = AC and AD = AE
 AB – AD = AC – AE
 DB = EC
Thus, we have
AD = AE and DB = EC

AD AE
 
DB EC
 DE || BC [By the converse of Thale’s Theorem]
 ABC = ADE [Corresponding angles]
 ABC + BDE = ADE + BDE [Adding BDE on both sides]
 ABC + BDE = 180º
 ACB + BDE = 180º [ AB = AC  ABC = ACB]
Again, DE || BC
 ACB = AED
 ACB + CED = AED + CED [Adding CED on both sides]
 ACB + CED = 180º
 ABC + CED = 180º [ ABC = ACB]
Thus, BDEC is quadrilateral such that ACB + BDE = 180º and ABC + CED = 180º

Ex.55 The side BC of a triangle ABC is bisected at D; O is any point in AD. BO and CO
produced meet AC and AB in E and F respectively and AD is produced to X so that O
is the mid-point of OX. Prove that AO : AX = AF : AB and show that FE || BC.
Sol. Join BX and CX. We have,
BD = CD and OD = DX.

FEAT Educational Ventures Pvt. Ltd.


H.O. 83, Heera Panna Mall, Near Jal-Vayu Vihar, Hiranandani Garden, Powai, Mumbai - 76 40
Call: +91-22-3226-8201 | SMS: +91-842-482-1664 | www.edventure.in | info@edventure.in
Foundation – IX Mathematics Geometry

Thus, BC and OX bisect each other.


 OB XC is a parallelogram.
 BX || CO and CX || BO
 BX || CF and CX || BE
 BX || OF and CX || OE
In ABX, we have
BX || OF
AO AF
  ----- (I)
AX AB
In ACX, we have
CX || OE
AO AE
  ----- (II)
AX AC
From equations (I), (II), we get
AF AE

AB AC
Thus, E and F are points on AB and AC such that they divide AB and AC respectively in the
same ratio. Therefore, by the converse of Thale’s Theorem FE || BC.

Ex.56 In fig., if DE || AQ and DF || AR. Prove that EF || QR.


Sol. In PQA, we have
DE || AQ [Given]

Therefore, by basic proportionality theorem, we have


PE PD
 ----- (I)
EQ DA
In PAR, we have
DF || AD [Given]
Therefore, by basic proportionality theorem, we have
PD PF
 ----- (II)
DA FR
From (I) and (II), we have
PE PF

DA FR

FEAT Educational Ventures Pvt. Ltd.


H.O. 83, Heera Panna Mall, Near Jal-Vayu Vihar, Hiranandani Garden, Powai, Mumbai - 76 41
Call: +91-22-3226-8201 | SMS: +91-842-482-1664 | www.edventure.in | info@edventure.in
Foundation – IX Mathematics Geometry

 EF || QR [By the converse of Basic Proportionality Theorem]

PS PT
Ex.57 In fig.,  and PST = PRQ. Prove that PQR is an isosceles triangle.
SQ TR
Sol. We have,
PS PT

SQ TR

 ST || QR [By using the converse of Basic Proportionally Theorem]


 PST = PQR [Corresponding angles]
 PRQ = PQR [ PST = PRQ (Given)]
 PQ = PR [ Side opposite to equal angles are equal]
 PQR is isosceles.

Ex.58 In fig., A, B and C are points on OP, OQ and OR respectively such that AB || PQ and
AC || PR. Show that BC || QR.
Sol. In OPQ, we have
AB || PQ

OA OB
  ----- (I)
AP BQ
In OQR, we have
BC || QR
OB OC
  ----- (II)
BQ CR
From (I) and (II), we get
OA OC

AP CR
Thus, A and C are points on sides OP and OR respectively of OPR, such that
OA OC

AP CR
 AC || PR [Using the converse of BPT]

FEAT Educational Ventures Pvt. Ltd.


H.O. 83, Heera Panna Mall, Near Jal-Vayu Vihar, Hiranandani Garden, Powai, Mumbai - 76 42
Call: +91-22-3226-8201 | SMS: +91-842-482-1664 | www.edventure.in | info@edventure.in
Foundation – IX Mathematics Geometry

Internal and external bisectors of an angle of a triangle

Theorem 1:
 The internal bisector of an angle of a triangle divides the opposite side internally in the ratio
of the sides containing the angle.

 Given: A ABC in which AD is the internal bisector of A and meets BC in D.

BD AB
 To Prove: 
DC AC

 Construction: Draw CE || DA to meet BA produced in E.

 Proof:
Since CE || DA and AC cuts them.
  2 = 3 ----- (I) [Alternate angles]
and, 1 = 4 ----- (II) [Corresponding angles]
But, 1 = 2 [ AD is the bisector of A]
From (I) and (II), we get
3 = 4
Thus, in ACE, we have
3 = 4
  AE = AC ----- (III) [Sides opposite to equal angles are equal]
Now, in BCE, we have
DA || CE
BD BA
  
DC AE
BD AB
   [ BA = AB and AE = AC (From (III))]
DC AC
BD AB
Hence, 
DC AC

Theorem 2:
BC AB
 In a triangle ABC, if D is a point on BC such that  , prove that AD is the bisector of
DC AC
A.

 In a triangle ABC, if D is a point on BC such that D divides BC in the ratio AB : AC, then AD
is the bisector of A.

FEAT Educational Ventures Pvt. Ltd.


H.O. 83, Heera Panna Mall, Near Jal-Vayu Vihar, Hiranandani Garden, Powai, Mumbai - 76 43
Call: +91-22-3226-8201 | SMS: +91-842-482-1664 | www.edventure.in | info@edventure.in
Foundation – IX Mathematics Geometry

 If a line through one vertex of a triangle divides the opposite sides in the ratio of other two
sides, then the line bisects the angle at the vertex.
BD AB
 Given: A ABC, in which D is a point on BC such that  .
DC AC

 To Prove: AD is the bisector of A.

 Construction: Produce BA to E such that AE = AC. Join EC.

 Proof:
In ACE, we have
AE = AC [By construction]
  3 = 4 ----- (I)
BD AB
Now, 
DC AC
BD AB
   [ AC = AE]
DC AE
Thus, in BCE, we have
BD AB
  
DC AE
Therefore, by the converse of Basic Proportionality
Theorem, we have
DA || CE
  1 = 4 ----- (II) [Corresponding angles]
and, 2 = 3 ----- (III) [Alternate angles]
But, 3 = 4 [From (I)]
  1 = 2 [From (II) and (III)]
Hence, AD is the bisector of A.

Theorem 3:
 The external bisector of an angle of a triangle divides the opposite side externally in the ratio
of the sides containing the angle.
 Given: A ABC, in which AD is the bisector of the exterior of angle A and intersects BC
produced
in D.

BD AB
 To Prove: 
CD AC

 Construction: Draw CE || DA meeting AB in E.

FEAT Educational Ventures Pvt. Ltd.


H.O. 83, Heera Panna Mall, Near Jal-Vayu Vihar, Hiranandani Garden, Powai, Mumbai - 76 44
Call: +91-22-3226-8201 | SMS: +91-842-482-1664 | www.edventure.in | info@edventure.in
Foundation – IX Mathematics Geometry

 Proof:
Since CE || DA and AC intersects them.
 1 = 3 ----- (I)
Also, CE || DA and BK intersects them.
 2 = 4 ----- (II)
But, 1 = 2 [ AD is the bisector of CAK 1  2 ]
 3 = 4 [From (I) and (II)]
Thus, in ACE, we have
 AE = AC ----- (III)
[ Sides opposite to equal angles in a  are equal]
Now, in BAD, we have
EC || AD
BD BA
  [Using corollary of Basic Proportionality Theorem]
CD EA
BD AB
  [ BA = AB and EA = AE]
CD AE
BD AB
  [ AE = AC, From (III)]
CD AC

Ex.59 In fig., AD is the bisector of A. If BD = 4cm, DC = 3 cm and AB = 6 cm, determine AC.
Sol. In ABC, AD is the bisector of A.
BD AB
 
DC AC
4 6
 
3 AC
 4AC  18
9
 AC  cm  4.5cm
2

Ex.60 In fig., AD is the bisector of BAC. If AB = 10 cm. AC = 14 cm and BC = 6 cm, find BD


and DC.
Sol. Let BD = x cm. Then, DC = (6 – x) cm.
Since AD is the bisector of A.

FEAT Educational Ventures Pvt. Ltd.


H.O. 83, Heera Panna Mall, Near Jal-Vayu Vihar, Hiranandani Garden, Powai, Mumbai - 76 45
Call: +91-22-3226-8201 | SMS: +91-842-482-1664 | www.edventure.in | info@edventure.in
Foundation – IX Mathematics Geometry

AB BD
 
AC DC
10 x
 
14 6  x
5 x
 
7 6x
 30 – 5x = 7x [ AC = 5.6]
 12x = 30
5
 x   2.5 cm
3
 BD = 2.5 cm and, DC = (6 – x) cm = (6 – 2.5) cm = 3.5 cm

Ex.61 The bisector of interior A of ABC meets BC in D, and the bisector of exterior A
BD CD
meets BC produced in E. Prove that  .
BE CE
Sol. Given: In ABC, AD and AE are respectively the bisectors of the interior and exterior angles
at A
BD CD
To Prove: 
BE CE
Proof: Since AD is the internal bisector of A meeting BC at D.

AB BD
  ----- (I)
AC DC
Since AE is the external bisector of A meeting BC produced in E.
AB BE
  ----- (II)
AC CE
BD BE
From (I) and (II), we get 
DC CE
BD CD
 
BE CE

FEAT Educational Ventures Pvt. Ltd.


H.O. 83, Heera Panna Mall, Near Jal-Vayu Vihar, Hiranandani Garden, Powai, Mumbai - 76 46
Call: +91-22-3226-8201 | SMS: +91-842-482-1664 | www.edventure.in | info@edventure.in
Foundation – IX Mathematics Geometry

Ex.62 If the diagonal BD of a quadrilateral ABCD bisects both B and D, show that
AB AD
 .
BC CD
Sol. Given: A quadrilateral ABCD in which the diagonal BD bisects B and D.
AB AD
To Prove: 
BC CD
Construction: Join AC intersecting BD in O.
Proof: In ABC, BO is the bisector of B.

AO BA
 
OC BC
OA AB
  ----- (I)
OC BC
In ADC, DO is the bisector of D.
AO DA
 
OC DC
OA AD
  ----- (II)
OC CD
AB AD
From (I) and (II), we get 
BC CD

Ex.63 ABCD is a quadrilateral in which AB = AD. The bisector of BAC and CAD intersect
the sides BC and CD at the points E and F respectively. Prove that EF || BD.
Sol. Given: A quadrilateral ABCD in which AB = AD and the bisectors of BAC and CAD meet
the sides BC and CD at E and F respectively.
To Prove: EF || BD
Construction: Join AC, BD and EF.

Proof: In CAB, AE is the bisector of BAC.


AC CE
  ----- (I)
AB BE
In ACD, AF is bisector of CAD.
AC CF
 
AD DF
AC CF
  [ AD = AB] ----- (II)
AB DF

FEAT Educational Ventures Pvt. Ltd.


H.O. 83, Heera Panna Mall, Near Jal-Vayu Vihar, Hiranandani Garden, Powai, Mumbai - 76 47
Call: +91-22-3226-8201 | SMS: +91-842-482-1664 | www.edventure.in | info@edventure.in
Foundation – IX Mathematics Geometry

From (I) and (II), we get


CE CF CE CF
  
BE DF EB FD
Thus, in CBD, E and F divide the sides CB and CD respectively in the same ratio.
Therefore, by the converse of Thale’s Theorem,
We have EF || BD.

Ex.64 O is any point inside a triangle ABC. The bisector of AOB, BOC and COA meet
the sides AB, BC and CA in point D, E and F respectively.
Show that AD  BE  CF  DB  EC  FA .
Sol. In AOB, OD is the bisector of AOB.

OA AD
  ----- (I)
OB DB
In BOC, OE is the bisector of BOC.
OB BE
  ----- (I)
OC EC
In COA, OF is the bisector of COA
OC CF
  ----- (III)
OA FA
Multiplying the corresponding sides of (i), (ii) and (iii), we get
OA OB OC AD BE CF
    
OB OC OA DB EC FA
AD BE CF
 1   
DB EC FA
 DB  EC  FA  AD  BE  CF
 AD  BE  CF  DB  EC  FA

Ex.65 AD is a median of ABC. The bisector of ADB and ADC meet AB and AC in E and F
respectively. Prove that EF || BC.
Sol. Given: In ABC, AD is the median and DE and DF are the bisectors of ADB and ADC
respectively, meeting AB and AC in E and F respectively.
To Prove: EF || BC
Proof: In ADB, DE is the bisector of ADB.

FEAT Educational Ventures Pvt. Ltd.


H.O. 83, Heera Panna Mall, Near Jal-Vayu Vihar, Hiranandani Garden, Powai, Mumbai - 76 48
Call: +91-22-3226-8201 | SMS: +91-842-482-1664 | www.edventure.in | info@edventure.in
Foundation – IX Mathematics Geometry

AD AE
  ----- (I)
DB EB
In ADC, DF is the bisector of ADC.
AD AF
 
DC FC
AD AF
  ----- (II) [ AD is the median BD  DC ]
DC FC
From (I) and (II), we get
AE AF

EB FC
Thus, in ABC, line segment EF divides the sides AB and AC in the same ratio.
Hence, EF is parallel to BC.

Ex.66 In a quadrilateral ABCD, if bisectors of the ABC and ADC meet on the diagonal AC,
rove that the bisectors of BAD and BCD will meet on the diagonal BD.
Sol. Given: ABCD is a quadrilateral in which the bisectors of ABC and ADC meet on the
diagonal AC at P.

To Prove: Bisectors of  BAD and BCD meet on the diagonal BD.


Construction: Join BP and DP, Let the bisector of BAD meet BD at Q. Join AQ and CQ.
Proof: In order to prove that the bisectors of BAD and BCD meet on the diagonal BD. It
is sufficient to prove that CQ is the bisector of BCD. For which we will prove that Q divides
BD in the ratio BC : DC.
In ABC, BP is the bisector of ABC.
AB AP
  ----- (I)
BC PC
In ACD, DP is the bisector of ADC.
AD AP
  ----- (II)
DC PA
From (I) and (II), we get
AB AD

BC DC

FEAT Educational Ventures Pvt. Ltd.


H.O. 83, Heera Panna Mall, Near Jal-Vayu Vihar, Hiranandani Garden, Powai, Mumbai - 76 49
Call: +91-22-3226-8201 | SMS: +91-842-482-1664 | www.edventure.in | info@edventure.in
Foundation – IX Mathematics Geometry

AB BC
  ----- (III)
AD DC
In ABD, AQ is the bisector of BAD [By construction]
AB BQ
  ----- (IV)
AD DQ
From (III) and (IV), we get
BC BQ
 
DC DQ
Thus, in CBD, Q divides BD in the ratio CB : CD. Therefore, CQ is the bisector of BCD.
Hence, bisectors of BAD and BCD meet on the diagonal BD.

Ex.67 If the bisector of an angle of a triangle bisects the opposite side, prove that the
triangle is isosceles.
Sol.

Given: In ABC, the bisector AD of A bisects the side BC,


To Prove: AB = AC
Proof: In ABC, AD is the bisector of A.
AB BD
 
AC DC
AB
  1. [D is mid-point of BC BD  DC ]
AC
 AB  AC
Hence, the triangle ABC is isosceles.

Ex.68 In figure, BAC = 90º, AD is its bisector.


If DE  AC, prove that DE   AB  AC   AB  AC .
Sol. It is given that AD is the bisector of A of ABC.

AB BD
 
AC DC
AB BC
 1 1 [Adding 1 on both sides]
AC DC

FEAT Educational Ventures Pvt. Ltd.


H.O. 83, Heera Panna Mall, Near Jal-Vayu Vihar, Hiranandani Garden, Powai, Mumbai - 76 50
Call: +91-22-3226-8201 | SMS: +91-842-482-1664 | www.edventure.in | info@edventure.in
Foundation – IX Mathematics Geometry

AB  AC BD  DC
 
AC DC
AB  AC BC
  ----- (I)
AC DC
In  ' s CDE and CBA, we have
DCE = BCA = C [Common]
BAC = DEC [Each equal to 90 ]
So, by A A-criterion of similarity, we have
CDE CBA
CD DE
 
CB BA
AB BC
  ----- (II)
DE DC
From (I) and (II), we have
AB  AC AB

AC DE
 DE   AB  AC   AB  AC

Ex.69 BO and CO are respectively the bisectors of B and C of ABC. AO produced meets
BC at P. Show that
AB AO AC AO
(I)  (II) 
BP OP CP OP
AB BP
(III)  (IV) AP is the bisector of BAC.
AC PC
Sol. (i) In ABP, BO is the bisector of B

AB AO
 
BP OP
(ii) In ACP, OC is the bisector of C.
AC AO
 
CP OP
(iii) We have, proved that
AB AO AC AO
 and 
BP OP CP OP
AB AC
 
BP CP
AB BP
 
AC PC
(iv) As proved above that in ABC, we have

FEAT Educational Ventures Pvt. Ltd.


H.O. 83, Heera Panna Mall, Near Jal-Vayu Vihar, Hiranandani Garden, Powai, Mumbai - 76 51
Call: +91-22-3226-8201 | SMS: +91-842-482-1664 | www.edventure.in | info@edventure.in
Foundation – IX Mathematics Geometry

AB BP
  AP is the bisector of BAC.
AC CP

Ex.70 Prove that the line drawn from the mid-point of one side of a triangle parallel of
another side bisects the third side.
Sol. Given: A ABC, in which D is the mid-point of side AB and the line DE is drawn parallel to
BC, meeting AC in E.
To Prove: E is the mid-point of AC i.e., AE = EC.
Proof: In ABC, we have
DE || BC
AD AE
  [By Thale’s Theorem] ----- (I)
DB EC

But, D is the mid-point of AB.


 AD  DB
AD
 1 ----- (II)
DB
From (I) and (II), we get
AE
1 
EC
  AE  EC
Hence, E bisects AC.

Ex.71 Prove that the line joining the mid-point of two sides of a triangle is parallel to the
third side.
Sol. Given: A ABC in which D and E are mid-point of sides AB and AC respectively.

To Prove: DE || BC
Proof: Since D and E are mid-points of AB and AC respectively.
 AD = DB and AE = EC
AE AE
  1 and 1
DB EC
AD AE
 
DB EC

FEAT Educational Ventures Pvt. Ltd.


H.O. 83, Heera Panna Mall, Near Jal-Vayu Vihar, Hiranandani Garden, Powai, Mumbai - 76 52
Call: +91-22-3226-8201 | SMS: +91-842-482-1664 | www.edventure.in | info@edventure.in
Foundation – IX Mathematics Geometry

Thus, the line DE divides the sides AB and AC of ABC in the same ratio. Therefore, by the
converse of Basic Proportionality Theorem, we have
DE || BC

Ex.72 Prove that the diagonals of a trapezium divide each other proportionally.
Sol. Given: A trapezium ABCD in which the diagonals AC and AD intersect at E.
DE CE
To Prove: 
EB EA
Construction: Draw EF || BA || CD, meeting AD in F.
Proof: In ABD, we have
FE || AB

DE DF
  [By Thale’s Theorem] ----- (I)
EB FA
In CDA, we have
FE || DC
CE DF
  [By Thale’s Theorem] ----- (II)
EA FA
From (I) and (II), we get
DE CE

EB EA

Ex.73 Prove that the diagonals of a quadrilateral divide each other proportionally, then it is a
trapezium.
Sol. Given: A quadrilateral ABCD whose diagonals AC and BD intersect at E such that
DE CE
 .
EB EA
To Prove: Quadrilateral ABCD is a trapezium. For this it is sufficient to prove that AB || DC.
Construction: Draw EF || BA, meeting AD in F.

Proof: In ABD, we have


FE || BA
DF DE
  [By Thale’s Theorem] ----- (I)
FA EB

FEAT Educational Ventures Pvt. Ltd.


H.O. 83, Heera Panna Mall, Near Jal-Vayu Vihar, Hiranandani Garden, Powai, Mumbai - 76 53
Call: +91-22-3226-8201 | SMS: +91-842-482-1664 | www.edventure.in | info@edventure.in
Foundation – IX Mathematics Geometry

DE CE
But,  [Given] ----- (II)
EB EA
From (I) and (II), we get
DF CE

FA EA
Thus, in DCA, E and F are points on CA and DA respectively such that
DF CE

FA EA
Therefore, by the converse of Basic Proportionality Theorem, we have
FE || DC
But, FE || BA [By construction]
 DC || BA
 AB || DC
Hence, ABCD is a trapezium.

Ex.74 Prove that any line parallel to the parallel sides of a trapezium divides the non-parallel
sides proportionally.
Sol. Given: A trapezium ABCD in which DC || AB and EF is a line parallel to DC and AB.
AE BF
To Prove: 
ED FC
Construction: Join AC, meeting EF in G.

Proof: In ADC, we have


EG || DC
AE AG
  [By Thale’s Theorem] ----- (I)
ED GC
In ABC, we have
GF || AB
AG BF
  [By Thale’s Theorem] ----- (II)
GC FC
From (I) and (II), we get
AE BF

ED FC

Ex.75 Prove that three or more parallel lines are intersected by two transversals, prove that
the intercepts made by them, on the transversals are proportional.
Sol. Given: Three parallel lines l, m, n which are cut by the transversals AB and CD in P, Q, R
and E, F, G respectively.

FEAT Educational Ventures Pvt. Ltd.


H.O. 83, Heera Panna Mall, Near Jal-Vayu Vihar, Hiranandani Garden, Powai, Mumbai - 76 54
Call: +91-22-3226-8201 | SMS: +91-842-482-1664 | www.edventure.in | info@edventure.in
Foundation – IX Mathematics Geometry

PQ EF
To Prove: 
QR FG
Construction: Draw PL || CD meeting the lines m and n in M and L respectively.
Proof: Since PE || MF and PM || EF.

 PMFE is a parallelogram
 PM = EF ----- (I)
Also, MF || LG and ML || FG.
 MLGF is a parallelogram
 ML = FG ----- (II)
Now, in PRL, we have
QM || RL
PQ PM
  [By Thale’s Theorem]
QR ML
PQ EF
  [Using (i) and (ii)]
QR FG
PQ EF
Hence, 
QR FG

Ex.76 Prove that the line segments joining the mid-points of the adjacent sides of a
quadrilateral form a parallelogram.
Sol. Given: A quadrilateral ABCD in which P, Q, R and S are the mid-point of sides AB, BC, CD
and DA respectively.
To Prove: PQRS is a parallelogram.
Construction: Join AC.
Proof: In ABC, P and Q are the mid-points of AB and BC respectively.

 PQ || AC ----- (I)
In ACD, R and S are the mid-points of CD and DA respectively.
 SR || AC
From (I) and (II), we have
PQ || AC and SR || AC
 PQ || SR
Similarly, by considering triangles ABD and BCD, we can prove that
PS || QR
Hence, PQRS is a parallelogram.

FEAT Educational Ventures Pvt. Ltd.


H.O. 83, Heera Panna Mall, Near Jal-Vayu Vihar, Hiranandani Garden, Powai, Mumbai - 76 55
Call: +91-22-3226-8201 | SMS: +91-842-482-1664 | www.edventure.in | info@edventure.in
Foundation – IX Mathematics Geometry

Ex.77 In figure, P is the mid-point of BC and Q is the mid-point of AP. If BQ when produced
1
meets AC at R, prove that RA  CA .
3
Sol. Given: A ABC in which P is the mid-point of BC, Q is the mid-point of AP, such that BQ
produced meets AC at R.
1
To Prove: RA  CA .
3
Construction: Draw PS || BR, meeting AC at S.
Proof: In BCR, P is the mid-point of BC and PS || SR.

 S is the mid-point of CR.


 CS = SR ----- (I)
In APS, Q is the mid-point of AP and QR || PS.
 R is the mid-point of AS.
 AR  RS ----- (II)
From (I) and (II), we get
AR = RS = SC
 AC  AR  RS  SC  3AR
1 1
 AB  AC  CA
3 3

Ex.78 In figure, AB || DC. Find the value of x.


Sol. Since the diagonals of a trapezium divide each other proportionally.

AO BO
  ----- (III)
OC OD
3x  19 x  3
 
x5 3
 3(3x – 19) = (x – 5) (x – 3)
2
 9x – 57 = x – 8x + 15
2
 x – 17x + 72 = 0
 (x – 8) (x – 9) = 0
 x – 8 = 0 or x – 9 = 0 
  x = 8 or x = 9

FEAT Educational Ventures Pvt. Ltd.


H.O. 83, Heera Panna Mall, Near Jal-Vayu Vihar, Hiranandani Garden, Powai, Mumbai - 76 56
Call: +91-22-3226-8201 | SMS: +91-842-482-1664 | www.edventure.in | info@edventure.in
Foundation – IX Mathematics Geometry

Criteria for similarity of triangles

Equiangular triangles
 Two triangles are said to be equiangular, if their corresponding angles are equal.

 If two triangles are equiangular, then they are similar.


Two triangles ABC and DEF such that 
A  D , B  E and C  F .
AB BC AC
Then ABC DEF and  
DE EF DF

Ex.79 Examine each pair of triangles in Figure, and state which pair of triangles are similar.
Also, state the similarity criterion used by you for answering the question and write
the similarity relation in symbolic form.

Figure (I) Figure (II) Figure (III)

Figure (IV) Figure (V) Figure (VI)


Sol. (I) In triangles ABC and PQR, we observe that
A = Q = 40º, B = P = 60º and C = R = 80º
Therefore, by AAA-criterion of similarity
ABC QRP or PQR BAC or ACB QRP
(II) In triangle PQR and DEF, we observe that
PQ QR PR 1
  
DE EF DF 2
Therefore, by SSS-criterion of similarity, we have
PQR DEF
(III) In triangles LMN and PQR, we have
M = P = 70º
MN ML
But, 
PQ PR
Therefore, these two triangles are not similar as they do not satisfy SAS-criterion of
similarity.
(IV) In ’s MNP and EFG, we observe that
NP MP MN
 
FG EG EF

FEAT Educational Ventures Pvt. Ltd.


H.O. 83, Heera Panna Mall, Near Jal-Vayu Vihar, Hiranandani Garden, Powai, Mumbai - 76 57
Call: +91-22-3226-8201 | SMS: +91-842-482-1664 | www.edventure.in | info@edventure.in
Foundation – IX Mathematics Geometry

Therefore, these two triangles are not similarly as they do not satisfy SSS-criterion of
similarity.
(V) In ’s ABC and DEF, we have
A = D = 80º
AB AC
But,  [ AC is not given]
DE DF
So, by SAS-criterion of similarity these two triangles are not similar.
(VI) In ’s DEF and MNP, we have
D = M = 70º
E = N = 80º [ N = 180º – M – P = 180º – 70º – 30º = 80º]
So, by AA-criterion of similarity
DEF MNP

Ex.80 In figure, find E.

Sol. In triangles ABC and DEF, we have


AB BC CA 1
  
DF EF ED 2
Therefore, by SSS-criterion of similarity, we have
ABC DEF
 A  D , B  F and C  E
 D  80 , F  60
Hence, F  60

Ex.81 In figure, ACB APQ . If BC = 8 cm, PQ = 4 cm, BA = 6.5 cm, AP = 2.8 cm, find CA
and AQ.
Sol. We have,
ACB APQ

AC CB AB
  
AP PQ AQ
AC CB CB AB
  and 
AP PQ PQ AQ
AC 8 8 6.5
  and 
2.8 4 4 AQ

FEAT Educational Ventures Pvt. Ltd.


H.O. 83, Heera Panna Mall, Near Jal-Vayu Vihar, Hiranandani Garden, Powai, Mumbai - 76 58
Call: +91-22-3226-8201 | SMS: +91-842-482-1664 | www.edventure.in | info@edventure.in
Foundation – IX Mathematics Geometry

AC 6.5
  2 and 2
2.8 AQ
 AC   2  2.8  cm  5.6 cm and
6.5
AQ  cm  3.25 cm
2

Ex.82 In figure, QA and PB are perpendicular to AB. If AO = 10 cm, BO = 6 cm and PB = 9


cm. Find AQ.
Sol. In triangles AOQ and BOP, we have
OAQ = OBP [Each equal to 90 ]
AOQ = BOP [Vertically opposite angles]
Therefore, by AA-criterion of similarity
AOQ BOP

AO OQ AO
  
BO OP BP
AO AQ 10 AQ
   
BO BP 6 9
10  9
 AQ   15 cm
6

Ex.83 In figure, CAB = 90º and AD  BC. If AC = 75 cm, AB = 1 m and BD = 1.25 m, find AD.
Sol. We have,
AB = 1m = 100 cm, AC = 75 cm and BD = 125 cm
In BAC and BDA, we have
BAC = BDA [Each equal to 90º]
and, B = B

So, by AA-criterion of similarity, we have


BAC BDA
BA AC
 
BD AD
100 75
 
125 AD

FEAT Educational Ventures Pvt. Ltd.


H.O. 83, Heera Panna Mall, Near Jal-Vayu Vihar, Hiranandani Garden, Powai, Mumbai - 76 59
Call: +91-22-3226-8201 | SMS: +91-842-482-1664 | www.edventure.in | info@edventure.in
Foundation – IX Mathematics Geometry

125  75
 AD  cm  93.75 cm
100

Ex.84 The perimeters of two similar triangles ABC and PQR are respectively 36 cm and 24
cm. If PQ = 10 cm, find AB.
Sol. Since the ratio of the corresponding sides of similar triangles is same as the ratio of their
perimeters.
 ABC PQR
AB BC AC 36
   
PQ QR PR 24
AB 36
 
PQ 24
AB 36
 
10 24
36  10
 AB  cm  15 cm
24

AO BO 1
Ex.85 In figure,   and AB = 5 cm. Find the value of DC.
OC OD 2
Sol. In AOB and COD, we have
AOB = COD [Vertically opposite angles]
AO OB
 [Given]
OC OD

So, by SAS-criterion of similarity, we have


AOB COD
AO BO AB
  
OC OD DC
1 5
  [ AB = 5 cm]
2 DC
 DC  10 cm

FEAT Educational Ventures Pvt. Ltd.


H.O. 83, Heera Panna Mall, Near Jal-Vayu Vihar, Hiranandani Garden, Powai, Mumbai - 76 60
Call: +91-22-3226-8201 | SMS: +91-842-482-1664 | www.edventure.in | info@edventure.in
Foundation – IX Mathematics Geometry

Ex.86 In figure, if A = C, then prove that AOB COD .

Sol. In triangles AOB and COD, we have


A = C [Given]
and, 1 = 2 [Vertically opposite angles]
Therefore, by AA-criterion of similarity, we have
AOB COD

CA CB
Ex.87 D is a point on the side BC of ABC such that ADC = BAC. Prove that  or,
CD CA
CA 2  CB  CD .
Sol. In ABC and DAC, we have
ADC = BAC and C = C

Therefore, by AA-criterion of similarity, we have


ABC DAC
AB BC AC
  
DA AC DC
CB CA
 
CA CD

Ex.88 In figure, considering triangles BEP and CPD, prove that BP  PD  EP  PC .


Sol. Given: A ABC in which BD  AC and CE  AB and BD and CE intersect at P.
To Prove: BP  PD  EP  PC
Proof:
In EPB and DPC, we have
PEB = PDC [Each equal to 90 ]
EPB = DPC [Vertically opposite angles]

FEAT Educational Ventures Pvt. Ltd.


H.O. 83, Heera Panna Mall, Near Jal-Vayu Vihar, Hiranandani Garden, Powai, Mumbai - 76 61
Call: +91-22-3226-8201 | SMS: +91-842-482-1664 | www.edventure.in | info@edventure.in
Foundation – IX Mathematics Geometry

Thus, by AA-criterion of similarity, we have


EPB DPC
FP PB

DP PC
 BP  PD  EP  PC

Ex.89 Two triangles BAC and BDC right angled at A and D respectively, are drawn on the
same base BC and on the same side of BC. If AC and DB intersect at P, prove that
AP  PC  DP  PB .
Sol. In APB and DPC, we have

A  D  90
and, APB = DPC [Vertically opposite angles]
Thus, by AA-criterion of similarity, we have
APB DPC
AP PB
  
DP PC
  AP  PC  DP  PB

Ex.90 P and Q are points on sides AB and AC respectively of ABC. If AP = 3 cm, PB = 6cm.
AQ = 5 cm and QC = 10 cm, show that BC = 3PQ.
Sol. We have,
AB = AP + PB = (3 + 6) cm = 9 cm
and, AC = AQ + QC = (5 + 10) cm = 15 cm.
AP 3 1 AQ 5 1
   and  
AB 9 3 AC 15 3
AP AQ
 
AB AC

Thus, in triangles APQ and ABC, we have

FEAT Educational Ventures Pvt. Ltd.


H.O. 83, Heera Panna Mall, Near Jal-Vayu Vihar, Hiranandani Garden, Powai, Mumbai - 76 62
Call: +91-22-3226-8201 | SMS: +91-842-482-1664 | www.edventure.in | info@edventure.in
Foundation – IX Mathematics Geometry

AP AQ
 and A = A [Common]
AB AC
Therefore, by SAS-criterion of similarity, we have
APQ ABC
AP PQ AQ
  
AB BC AC
PQ AQ PQ 5
   
BC AC BC 15
PQ 1
   BC  3PQ
BC 3

Ex.91 In figure, express x in terms of a, b and c.


Sol. In KPN and KLM, we have

KNP  KML  46 [Given]


K = K [Common]
 KNP KML [By AA-criterion of similarity]
KN NP
  [ Corresponding sides of similar triangles are proportional]
KM ML
c x ac
   x
bc a bc

Ex.92 Two poles of height a metres and b metres are p metres apart. Prove that the height of
the point of intersection of the lines joining the top of each pole to the foot of the
ab
opposite pole is given by metres.
ab
Sol. Let AB and CD be two poles of heights a metres and b metres respectively such that the
poles are p metres apart i.e. AC = p metres. Suppose the lines AD and BC meet at O such
that OL = h metres.
Let CL = x and LA = y. Then, x  y  p .

In ABC and LOC, we have


CAB = CLO [Each equal to 90 ]
C = C [Common]
 CAB CLO [By AA-criterion of similarity]

FEAT Educational Ventures Pvt. Ltd.


H.O. 83, Heera Panna Mall, Near Jal-Vayu Vihar, Hiranandani Garden, Powai, Mumbai - 76 63
Call: +91-22-3226-8201 | SMS: +91-842-482-1664 | www.edventure.in | info@edventure.in
Foundation – IX Mathematics Geometry

CA AB
 
CL LO
P a
 
x h
ph
 x ----- (I)
a
In ALO and ACD, we have
ALO = ACD [Each equal to 90 ]
A = A [Common]
 ALO ACD [By AA-criterion of similarity]
AL OL
 
AC DC
p a
 
x h
ph
 y  AC  x  y  p  ----- (II)
b
From (I) and (II), we have
ph ph
xy 
a b
 1 1
 p  ph     x  y  p
a b
ab
 1  h 
 ab 
ab
 h metres.
ab
Hence, the height of the intersection of the lines joining the top of each pole to the foot of the
ab
opposite pole is metres.
ab

Ex.93 ABC is a triangle in which AB = AC and D is a point on AC such that BC2  AC  CD .


Prove that BD = BC.
Sol. Given: ABC in which AB = AC and D is a point on the side AC such that BC2  AC  CD

To Prove: BD = BC
Construction: Join BD
Proof: We have,
BC2  AC  CD
BC AC
  ----- (I)
CD BC

FEAT Educational Ventures Pvt. Ltd.


H.O. 83, Heera Panna Mall, Near Jal-Vayu Vihar, Hiranandani Garden, Powai, Mumbai - 76 64
Call: +91-22-3226-8201 | SMS: +91-842-482-1664 | www.edventure.in | info@edventure.in
Foundation – IX Mathematics Geometry

Thus, in ABC and BDC, we have


AC BC
 [From (I)]
BC CD
and, C = C [Common]
 ABC BDC [By SAS criterion of similarity]
AB BC
 
BD DC
AC BC
 
BD CD
AC BD
 
BC CD
From (I) and (II), we get
BC BD
  BD  BC
CD CD

Ex.94 In figure, BAC  90 and segment AD  BD . Prove that AD2  BD  DC .


Sol. In ABD and ACD, we have

ADB = ADC [Each equal to 90 ]


and, DBA = DAC [Each equal to complement of BAD i.e., 90  BAD ]
Therefore, by AA-criterion of similarity, we have
DBA DAC  D  D,   DAC and BAD  DCA 
DB DA
 
DA DC
BD AD
   AD2  BD  DC
AD DC

Ex.95 The diagonal BD of a parallelogram ABCD intersects the segment AE at the point F,
where E is any point on the side BC. Prove that DF  EF  FB  FA .
Sol. In AFD and BFE, we have
1 = 2 [Vertically opposite angles]
3 = 4 [Alternate angles]

So, by AA-criterion of similarity, we have


FBE FDA

FEAT Educational Ventures Pvt. Ltd.


H.O. 83, Heera Panna Mall, Near Jal-Vayu Vihar, Hiranandani Garden, Powai, Mumbai - 76 65
Call: +91-22-3226-8201 | SMS: +91-842-482-1664 | www.edventure.in | info@edventure.in
Foundation – IX Mathematics Geometry

FB FD FB EF
   
FD FA DF FA
 DF  EF  FB  FA

Ex.96 In a ABC, BD and CE are the altitudes. Prove that ADB and AEC are similar. Is
CDB BEC ?
Sol. In ABD and AEC, we have
ADB = AEC [Each equal to 90 ]
BAD = EAC

So, by AA-criterion of similarity, we have


BDA CEA or ADB AEC .
Clearly, CDB is not similar to BEC, because they are not equiangular.

Ex.97 In figure, ABCD is a trapezium with AB || DC. If AED is similar to BEC, prove that
AD  BC .
Sol. In EDC and EBA, we have
1 = 2 [Alternate angles]
3 = 4 [Alternate angles]
and, CED = AEB [Vertically opposite angles]
 EDC EBA

ED EC
 
EB EA
ED EB
  ----- (I)
EC EA
It is given that AED BEC
ED EA AD
   ----- (II)
EC EB BC
From (I) and (II), we get
EB EA

EA EB
EB  EA 
2 2

FEAT Educational Ventures Pvt. Ltd.


H.O. 83, Heera Panna Mall, Near Jal-Vayu Vihar, Hiranandani Garden, Powai, Mumbai - 76 66
Call: +91-22-3226-8201 | SMS: +91-842-482-1664 | www.edventure.in | info@edventure.in
Foundation – IX Mathematics Geometry

 EB = EA
Substituting EB = EA in (II), we get
EA AD

EA BC
AD
 1
BC
 AD = BC

Ex.98 Through the mid-point M of the side CD of a parallelogram ABCD, the line BM is
drawn intersecting AC in L and AD produced in E. Prove that EL = 2 BL.
Sol. In BMC and EMD, we have
MC = MD [ M is the mid-point of CD]
CMB = EMD [Vertically opposite angles]
and, MBC = MED [Alternate angles]
So, by AAS-criterion of congruence, we have
 BMC  EMD
 BC  DE ----- (I)
Also, AD = BC ----- (II) [ ABCD is a parallelogram]
AD  DE  BC  BC
 AE  2BC ----- (III)
Now, in AEL and CBL, we have
ALE = CLB [Vertically opposite angles]
EAL = BCL [Alternate angles]
So, by AA-criterion of similarity of triangles, we have

AEL CBL
EL AE
 
BL CB
FL 2BC
  [Using equations (III)]
BL BC
EL
 2
BL
 EL  2BL

FEAT Educational Ventures Pvt. Ltd.


H.O. 83, Heera Panna Mall, Near Jal-Vayu Vihar, Hiranandani Garden, Powai, Mumbai - 76 67
Call: +91-22-3226-8201 | SMS: +91-842-482-1664 | www.edventure.in | info@edventure.in
Foundation – IX Mathematics Geometry

Ex.99 In figure, FEC  GBD and 1 = 2. Prove that ADE ABC .

Sol. We have,
FEC  GBD
 EC = BD ----- (I)
It is given that
1 = 2
 AD = AE ----- (II)
From (I) and (II), we have
AE AD

EC BD
 DE || BC [By the converse of basic proportionality theorem]
 1 = 3 and 2 = 4
Thus, in ’s ADE and ABC, we have
A = A
1 = 3
2 = 4
So, by AAA-criterion of similarity, we have
ADE ABC

Ex.100 In a ABC, let P and Q be points on AB and AC respectively such that PQ || BC. Prove
that median AD bisects PQ.
Sol. Suppose the median AD intersects PQ at E.
Now, PQ || BC [Corresponding angles]

 APE = B and AQE = C


So, in ’s APE and ABD, we have
APE = ABD
and, PAE = BAD [Common]
 APE ABD
PE AE
  ----- (I)
BD AD
Similarly, we have
AQE ACD

FEAT Educational Ventures Pvt. Ltd.


H.O. 83, Heera Panna Mall, Near Jal-Vayu Vihar, Hiranandani Garden, Powai, Mumbai - 76 68
Call: +91-22-3226-8201 | SMS: +91-842-482-1664 | www.edventure.in | info@edventure.in
Foundation – IX Mathematics Geometry

QE AE
  ----- (II)
CD AD
From (I) and (II), we get
PF QE

BD CD
PE QE
  [ AD is the median  BD = CD]
BD BD
 PE = QE
Hence, AD bisects PQ.

Ex.101 In figure, DEFG is a square and BAC  90 , Prove that


(I) AGF DBG (II) AGF EFC
(III) DBG EFC (IV) DE2  BD  EC
Sol. (i) In AGF and DBG, we have

GAF = BDG [Each equal to 90 ]


and, AGF = DBG [Corresponding angles]
 AGF DBG [By AA-criterion of similarity]
(ii) In AGF and EFC, we have
FAG = CEF [Each equal to 90 ]
and, AFG = ECF
 DBG EFC [By AA-criterion of similarity]
(iii) Since AGF DBG and AGE EFC
 DBG EFC
(iv) We have,
DBG EFC [Using (III)]
BD DG
 
EF EC
BD DE
  [ DEFG is a square  EF = DE, DG = DE]
DE EC
 DE2  BD  BC

Ex.102 If a perpendicular is drawn from the vertex containing the right angle of a right
triangle to the hypotenuse then prove that the triangle on each side of the
perpendicular are similar to each other and to the original triangle. Also, prove that
the square of the perpendicular is equal to the product of the lengths of the two parts
of the hypotenuse.
Sol. Given: A right triangle ABC right angled at B, BD  AC.
To Prove:

FEAT Educational Ventures Pvt. Ltd.


H.O. 83, Heera Panna Mall, Near Jal-Vayu Vihar, Hiranandani Garden, Powai, Mumbai - 76 69
Call: +91-22-3226-8201 | SMS: +91-842-482-1664 | www.edventure.in | info@edventure.in
Foundation – IX Mathematics Geometry

(i) ADB BDC (ii) ADB ABC


(iii) BDC ABC (iv) BD  AD  DC
2

(v) AB  AD  AC
2
(vi) BC2  CD  AC
Proof:

(i) We have,
ABD + DBC = 90º
Also, C + DBC + BDC = 180º
 C + DBC + 90º = 180º
 C + DBC = 90º
But, ABD + DBC = 90º
 ABD + DBC = C + DBC
 ABD = C ----- (I)
Thus, in ADB and BDC, we have
ABD = C [From (I)]
and, ADB = BDC [Each equal to 90º]
So, by AA-similarity criterion, we have
ADB BDC
(ii) In ADB and ABC, we have
ADB = ABC [Each equal to 90º]
and, A = A [Common]
So, by AA-similarity criterion, we have
ADB ABC
(iii) In BDC and ABC, we have
BDC = ABC [Each equal to 90º]
C = C [Common]
So, by AA-similarity criterion, we have
BDC ABC
(iv) From (i), we have
ADB BDC
AD BD
   BD2  AD  DC
BD DC
(v) From (ii), we have
ADB ABC
AD AB
   AB2  AD  AC
AB AC
(vi) From (iii), we have
BDC ABC
BC DC
   BC2  CD  AC
AC BC

FEAT Educational Ventures Pvt. Ltd.


H.O. 83, Heera Panna Mall, Near Jal-Vayu Vihar, Hiranandani Garden, Powai, Mumbai - 76 70
Call: +91-22-3226-8201 | SMS: +91-842-482-1664 | www.edventure.in | info@edventure.in
Foundation – IX Mathematics Geometry

Ex.103 Prove that the line segments joining the mid points of the sides of a triangle form four
triangles, each of which is similar to the original triangle.
Sol. Given:ABC in which D, E, F are the mid-points of sides BC, CA and AB respectively.
To Prove: Each of the triangles AFE, FBD, EDC and DEF is similar to ABC.
Proof: Consider triangles AFE and ABC.
Since F and E are mid-points of AB and AC respectively.
 FE || BC
 AEF = B [Corresponding angles]
Thus, in AFE and ABC, we have

AFE = B
and, A = A [Common]
 AEF ABC
Similarly, we have
FBD ABC and EDC ABC
Now, we shall show that DEF ABC
Clearly, ED || AF and DE || EA.
 AFDE is a parallelogram.
 EDF = A [ Opposite angles of a parallelogram are equal]
Similarly, BDEF is a parallelogram.
 DEF = B [ Opposite angles of a parallelogram are equal]
Thus, in triangles DEF and ABC, we have
EDF = A and DEF = B
So, by AA-criterion of similarity, we have
DEF ABC
Thus, each one of the triangles AFE, FBD, EDC and DEF is similar to ABC.

AD 2 BC
Ex.104 In ABC, DE is parallel to base BC, with D on AB and E on AC. If  , find .
DB 3 DE
Sol. In ABC, we have

AB AC
DE || BC  
AD AE
Thus, in triangles ABC and ADE, we have

FEAT Educational Ventures Pvt. Ltd.


H.O. 83, Heera Panna Mall, Near Jal-Vayu Vihar, Hiranandani Garden, Powai, Mumbai - 76 71
Call: +91-22-3226-8201 | SMS: +91-842-482-1664 | www.edventure.in | info@edventure.in
Foundation – IX Mathematics Geometry

AB AC
 and, A = A
AD AE
Therefore, by SAS-criterion of similarity, we have
ABC ADE
AD BC
  ----- (I)
AD DE
It is given that
AD 2 DB 3
  
DB 3 AD 2
DB 3
 1 1
AD 2
DB  AD 5
 
AD 2
AB 5
  ----- (II)
DE 2
From (I) and (II), we get
BC 5

DE 2

Ex.105 In figure, if ABE  ACD , prove that ADE ABC .


Sol. It is given that
ABE ACD

 AB  AC [ Corresponding parts of congruent triangles are equal]


and, AE = AD
AB AC
 
AD AE
AB AD
  ----- (I)
AC AE
Thus, in triangles ADE and ABC, we have
AB AD

AC AE
and BAC = DAE [Common]
Hence, by SAS-criterion of similarity, we have
ADE ABC

FEAT Educational Ventures Pvt. Ltd.


H.O. 83, Heera Panna Mall, Near Jal-Vayu Vihar, Hiranandani Garden, Powai, Mumbai - 76 72
Call: +91-22-3226-8201 | SMS: +91-842-482-1664 | www.edventure.in | info@edventure.in
Foundation – IX Mathematics Geometry

Ex.106 Through the mid-point M of the side CD of a parallelogram ABCD, the line BM is
drawn intersecting AC at L and AD produced at E. Prove that EL = 2 BL.
Sol. In ’s BMC and EMD, we have

BMC = EMD [Vertically opposite angles]


MC = MD [ M is the mid-point of CD]
MCB = MDE [Alternate angles]
So, by AAS-congruence criterion, we have
BMC  EMD
 BC = ED [ Corresponding parts of congruent triangle are equal]
In ’s AEL and CBL, we have
ALE = CLB [Vertically opposite angles]
EAL = BCL [Alternate angles]
So, By AA-criterion of similarity, we have
AEL CBL
AE EL AL
  
BC BL CL
EL AE
 
BL BC
EL AD  DE BC  DE 2BC
    2 [ AD = BC and DE = BC]
BL BC BC BC
 EL = 2BL.

More on characteristic properties

Theorem 1
 If two triangles are equiangular, prove that the ratio of the corresponding sides is same as
the ratio of the corresponding medians.

 Given: Two triangles ABC and DEF in which A = D, B = E, C = F, AP and DQ are
their medians.

Figure (I) Figure (II)

FEAT Educational Ventures Pvt. Ltd.


H.O. 83, Heera Panna Mall, Near Jal-Vayu Vihar, Hiranandani Garden, Powai, Mumbai - 76 73
Call: +91-22-3226-8201 | SMS: +91-842-482-1664 | www.edventure.in | info@edventure.in
Foundation – IX Mathematics Geometry

BC AP
 To Prove: 
EF DQ

 Proof:
Since equiangular triangles are similar.
 ABC DEF
AB BC
 
DE EF
AB 2BP
  [ P and Q are mid-point of BC and EF respectively.
DE 2EQ
 BC = 2BP and EF = 2EQ]
AB BP
  ----- (II)
DE EQ
Now, in ABP and DFQ, we have
AB BP
 [From (ii)]
DE EQ
and, B = E [Given]
So, by SAS-criterion of similarity, we have
ABP DEQ
AB AP
  ----- (III)
DE DQ
From (I) and (III), we get
BC AP

EF DQ
Hence, the ratio of the corresponding sides is same as the ratio of corresponding medians.

Theorem 2
 If two triangles are equiangular, prove that the ratio of the corresponding sides is same as
the ratio of the corresponding angle bisector segments.

 Given: Two triangles ABC and DEF in which A = D, B = E, C = F; and AX, DY are
the bisectors of A and D respectively.

Figure (I) Figure (II)

BC AX
 To Prove: 
EF DY

 Proof: Since equiangular triangles are similar.


  ABC DEF

FEAT Educational Ventures Pvt. Ltd.


H.O. 83, Heera Panna Mall, Near Jal-Vayu Vihar, Hiranandani Garden, Powai, Mumbai - 76 74
Call: +91-22-3226-8201 | SMS: +91-842-482-1664 | www.edventure.in | info@edventure.in
Foundation – IX Mathematics Geometry

AB BC
  ----- (I)
DE EF
In ABX and DEY, we have
B = E [Given]
 1 1 
and, BAX = EDY  A  D  A  D  BAX  EDY 
 2 2 
So, by AA-criterion of similarity, we have
ABX DEY
AB AX
  ----- (II)
DE DY
From (I) and (II), we get
BC AX

EF DY

Theorem 3
 If two triangles are equiangular, prove that the ratio of the corresponding sides is same as
the ratio of the corresponding altitudes.

 Given: Two triangles ABC and DEF in which A = D, B = E, C = F and AL  BC,
DM  EF

BC AL
 To Prove: 
EF DM

 Proof:
Since equiangular triangles are similar.
 ABC DEF
AB BC
  ----- (I)
DE EF
In triangle ALB and DME, we have
ALB = DME [Each equal to 90 ]
  B = E [Given]
So, by AA-criterion of similarity, we have
ALB DME
AB AL
  ----- (II)
DE DM
From (I) and (II), we get
BC AL

EF DM

FEAT Educational Ventures Pvt. Ltd.


H.O. 83, Heera Panna Mall, Near Jal-Vayu Vihar, Hiranandani Garden, Powai, Mumbai - 76 75
Call: +91-22-3226-8201 | SMS: +91-842-482-1664 | www.edventure.in | info@edventure.in
Foundation – IX Mathematics Geometry

Theorem 4
 If one angle of a triangle is equal to one angle of another triangle and the bisectors of these
equal angles divide the opposite side in the same ratio, prove that the triangles are similar.

 Given: Two triangles ABC and DEF in which A = D. The bisectors AP and DQ or A and
BP EQ
D intersect BC and EF in P and Q respectively such that  .
PC QF

 To Prove: ABC DEF

 Proof:
We know that the bisectors of an angle of triangle of a triangle intersect the opposite side in
the ratio of the sides containing the angle.

 AP is the bisector of A
BP AB
  ----- (I)
PC AC
DQ is the bisector of D
EQ DE
  ----- (II)
QF DF
BP EQ
But,  [Given]
PC QF
Therefore, from (I) and (II), we get
AB DE

AC DF
Thus, in triangles ABC and DEF, we have
AB DE

AC DF
and, A = D [Given]
So, by SAS-criterion of similarity, we get
ABC DEF

Theorem 5
 If two sides and a median bisecting one of these sides of a triangle are respectively
proportional to the two sides and the corresponding median of another triangle, then the
triangles are similar.

AB BC AP
 Given: ABC and DEF in which AP and DQ are the medians such that  
DE EF DQ

 To Prove: ABC DEF

FEAT Educational Ventures Pvt. Ltd.


H.O. 83, Heera Panna Mall, Near Jal-Vayu Vihar, Hiranandani Garden, Powai, Mumbai - 76 76
Call: +91-22-3226-8201 | SMS: +91-842-482-1664 | www.edventure.in | info@edventure.in
Foundation – IX Mathematics Geometry

 Proof:
AB BC AP
We have,  
DE EF DQ

1
BC
AB 2 AP
  
DE 1 DQ
EF
2
AB BP AP
  
DE EQ DQ
 ABP DEQ [By SSS-similarity]
 B = E
Now, in ABC and DEF, we have
AB BC
 [Given]
DE EF
and, B = E
So, by SAS-criterion of similarity, we get
ABC DEF

Theorem 6
 If two sides and a median bisecting the third side of a triangle are respectively proportional to
the corresponding sides and the median of another triangle, then the two triangles are
similar.

 Given: Two triangle ABC and DEF in which AP and DQ are the medians such that
AB AC AP
  .
DE DF DQ

 To Prove: ABC DEF

 Construction: Produce AP to G so that PG = AP. Join CG. Also, produce DQ to H so that


QH  DQ . Join FH.

 Proof:
In APB and GPC, we have
BP = CP [ AP is the median]
AP = GP [By construction]
and, APB = CPG [Vertically opposite angles]
So, by SAS-criterion of congruence, we have
  APB  GPC
  AG = GC ----- (I)

FEAT Educational Ventures Pvt. Ltd.


H.O. 83, Heera Panna Mall, Near Jal-Vayu Vihar, Hiranandani Garden, Powai, Mumbai - 76 77
Call: +91-22-3226-8201 | SMS: +91-842-482-1664 | www.edventure.in | info@edventure.in
Foundation – IX Mathematics Geometry

Again, In DQE and HQF, we have


EQ = FQ [ DQ is the median]
DQ = HQ [By construction]
and, DQE = HQF [Vertically opposite angles]
So, by SAS-criterion of congruence, we have
  DQE  HQF
  DE = HF ----- (II)
AB AC AP
Now,   [Given]
DE DF DQ
GC AC AP
    [ AB = GC and DE = HF (from (I) and (II))]
HF DF DQ
GC AC 2AP
   
HF DF 2DQ
GC AC AG
    [ 2AP = AG and 2DQ = DH]
HF DF DH
  AGC DHF [By SSS-criterion of similarity]
  1 = 2
Similarly, we have
  3 = 4
  1 3  2  4
  A = D ----- (III)
Thus, in ABC and DEF, we have
  A = D [From (III)]
AB AC
and,  [Given]
DE DF
So, by SAS-criterion of similarity, we have
  ABC DEF

Areas of two similar triangles

Theorem 1
 The ratios of the areas of two similar triangles are equal to the ratio of the squares of any
two corresponding sides.

 Given: Two triangles ABC and DEF such that ABC DEF .

FEAT Educational Ventures Pvt. Ltd.


H.O. 83, Heera Panna Mall, Near Jal-Vayu Vihar, Hiranandani Garden, Powai, Mumbai - 76 78
Call: +91-22-3226-8201 | SMS: +91-842-482-1664 | www.edventure.in | info@edventure.in
Foundation – IX Mathematics Geometry

Area  ABC AB2 BC2 AC2


 To Prove:   
Area  DEF DE2 EF2 DF2

 Construction: Draw AL  BC and DM  EF.

 Proof:
Since similar triangles are equiangular and their corresponding sides are proportional. Therefore,
  ABC DEF
  A = D, B = E, C = F
AB BC AC
and   ----- (I)
DE EF DF
Thus, in ALB and DME, we have
  ALB = DME [Each equal to 90º]
and, B = E [From (I)]
So, by AA-criterion of similarity, we have
ALB DME
AL AB
   ----- (II)
DM DE
From (I) and (II), we get
AB BC AC AL
   ----- (II)
DE EF DF DM
Area  ABC 
Now,
Area  DEF 
1
Area  ABC  BC  AL 
  2
Area  DEF  1
EF  DM
2
Area  ABC BC AL
  
Area  DEF  EF DM
Area  ABC BC BC BC AL
   [From (III).  ]
Area  DEF  EF EF EF DM
Area  ABC BC2
 
Area  DEF EF2
BC AB AC
But,  
EF DE DF
BC2 AB2 AC2
  
EF2 DE2 DF2
Area  ABC AB2 BC2 AC2
Hence,   
Area  DEF DE2 EF2 DF2

FEAT Educational Ventures Pvt. Ltd.


H.O. 83, Heera Panna Mall, Near Jal-Vayu Vihar, Hiranandani Garden, Powai, Mumbai - 76 79
Call: +91-22-3226-8201 | SMS: +91-842-482-1664 | www.edventure.in | info@edventure.in
Foundation – IX Mathematics Geometry

Theorem 2
 The areas of two similar triangles are in the ratio of the squares of the corresponding
altitudes.

 Given: Two triangles ABC and DEF such that ABC DEF and AL  BC, DM  EF then

Area  ABC AL2



Area  DEF DM2

Theorem 3
 The areas of two similar triangles are in the ratio of the squares of the corresponding
medians.

 Given: Two triangles ABC and DEF such that ABC DEF and AP, DQ are their medians,
Area  ABC AP2
then 
Area  DEF DQ2

Theorem 4
 The areas of two similar triangles are in the ratio of the squares of the corresponding angle
bisector segments.

 Given: ABC DEF and AX and DY are bisector of A and D respectively. Then

Area  ABC AX2



Area  DEF DY2

FEAT Educational Ventures Pvt. Ltd.


H.O. 83, Heera Panna Mall, Near Jal-Vayu Vihar, Hiranandani Garden, Powai, Mumbai - 76 80
Call: +91-22-3226-8201 | SMS: +91-842-482-1664 | www.edventure.in | info@edventure.in
Foundation – IX Mathematics Geometry

Theorem 5
 If the areas of two similar triangles are equal, then the triangles are congruent i.e. equal and
similar triangles are congruent.

 Given: Two triangles ABC and DEF such that ABC DEF and Area (ABC) = Area
(DEF).

 To Prove: We have, ABC  DEF

 Proof:
ABC DEF
AB BC AC
  A = D, B = E, C = F and  
DE EF DF
In order to prove that ABC  DEF , it is sufficient to show that AB = DE, BC = EF and
AC  DF .
Now, Area (ABC) = Area (DEF)
Area  ABC
  1
Area  DEF 

AB2 BC2 AC2  Area  ABC AB2 BC2 AC2 


    1     
DE2 EF2 DF2  Area  DEF  DE2 EF2 DF2 
 AB2  DE2 , BC2  EF2 and AC2  DF2
  AB = DE, BC = EF and AC = DF
Hence, ABC  DEF .

Ex.107 If Δ ABC : Δ DEF such that AB = 1.2 cm and DE = 1.4 cm. Find the ratio of area of
Δ ABC and Δ DEF .
Sol. We know that the ratio of areas of two similar triangles is equal to the ratio of the squares of
any two corresponding sides.
Area ( ABC) AB2
 
Area ( DEF) DF2

Area ( ABC) 1.2 


2 2
 12  36
    
Area ( DEF) 1.4  2
 14  49

Ex.108 In two similar triangles ABC and PQR, if their corresponding altitudes AD and PS are
in the ratio 4 : 9, find the ratio of the areas of ΔABC and ΔPQR .
Sol. Since the areas of two similar triangles are in the ratio of the squares of the corresponding
altitudes.
Area ( ABC) AD2
 
Area ( PQR) PS2
2
Area ( ABC)  4  16
   
Area ( PQR)  9  81
 AD: PS  4 : 9

FEAT Educational Ventures Pvt. Ltd.


H.O. 83, Heera Panna Mall, Near Jal-Vayu Vihar, Hiranandani Garden, Powai, Mumbai - 76 81
Call: +91-22-3226-8201 | SMS: +91-842-482-1664 | www.edventure.in | info@edventure.in
Foundation – IX Mathematics Geometry

Hence, Area  ABC  : Area  PQR  = 16 : 81

Ex.109 If ΔABC is similar to ΔDEF such that BC = 3 cm, EF = 4 cm and area of ΔABC = 54
cm. Determine the area of ΔDEF .
Sol. Since the ratio of areas of two similar triangles is equal to the ratio of the squares of any two
corresponding sides.
Area ( ABC) BC2
 
Area ( DEF) EF2
54 32
  2
Area ( DEF) 4
54  16
 Area  DEF   96 cm2
9

Ex.110 If ΔABC ΔDEF such that area of ΔABC is 9cm 2 and the area of ΔDEF is 16cm 2
and BC = 2.1 cm. Find the length of EF.
Sol. We have,
Area (ABC) BC2

Area ( DEF) EF2
9 (2.1)2
 
16 EF2
3 2.1 4  2.1
   EF  cm  2.8cm
4 EF 3

Ex.111 In fig., PB and QA are perpendiculars to segment AB. If PO = 5 cm, QO = 7 cm and


Area ΔPOB = 150 cm 2 find the area of ΔQOA .
Sol. In OAQ and OBP , we have
A  B Each equal to 90
AOQ  BOP

So, by AA-criterion of similarly, we have


AOQ  BOP
Area ( AOQ) OQ2
 
Area ( BOP) OP2
Area ( AOQ) 72
  2
150 5
49
 Area  AOQ   150 cm2  294 cm2
25

FEAT Educational Ventures Pvt. Ltd.


H.O. 83, Heera Panna Mall, Near Jal-Vayu Vihar, Hiranandani Garden, Powai, Mumbai - 76 82
Call: +91-22-3226-8201 | SMS: +91-842-482-1664 | www.edventure.in | info@edventure.in
Foundation – IX Mathematics Geometry

Ex.112 Prove that the area of the triangle BCE described on one side BC of a square ABCD as
base is one half the area of the similar triangle ACF described on the diagonal AC as
base.
Sol. ABCD is a square. BCE is described on side BC is similar to ACF described on diagonal
AC.
Since ABCD is a square. Therefore, AB = BC = CD = DA and, AC = 2 BC
 Diagonal  2 Side 
 

Now, BCE ~ ACF


Area ( BCE) BC2
 
Area ( ACF) AC2
Area ( BCE) BC2 1
  
Area ( ACF)
 
2
2 BC 2

 Area  BCE  = Area  ACF 

Ex.113 Prove that the area of the equilateral triangle described on the side of a square is half
the area of the equilateral triangle described on its diagonal.
Sol. Given: A square ABCD. Equilateral triangles BCE and ACF have been described on
side BC and diagonal AC respectively.
To Prove: Area  BCE   Area  ACF 
1
2

Proof: Since BCE and ACT are equilateral. Therefore, they are equiangular (each
angle being equal to 60º) and hence
BCE ~ ACF
Area ( BCE) BC2
 
Area ( ACF) AC2

Area ( BCE) BC2  ABCD is a square 


   
Area ( ACF)
   diagonal  2  side   AC  2BC
2
2BC

Area  BCE  1
 
Area  ACF  2

FEAT Educational Ventures Pvt. Ltd.


H.O. 83, Heera Panna Mall, Near Jal-Vayu Vihar, Hiranandani Garden, Powai, Mumbai - 76 83
Call: +91-22-3226-8201 | SMS: +91-842-482-1664 | www.edventure.in | info@edventure.in
Foundation – IX Mathematics Geometry

Ex.114 Equilateral triangles are drawn on the sides of a right triangle. Show that the area of
the triangle on the hypotenuse is equal to the sum of the areas of triangles on the
other two sides.
Sol. Given: A right angled ABC with right angle at B. Equilateral triangles PAB, QBC and RAC
are described on sides AB, BC and CA respectively.
To Prove: Area  PAB   Area  QBC   Area  RAC 

Proof: Since triangles PAB, QBC and RAC are equilateral, Therefore, they are equiangular
and hence similar.
Area ( PAB) Area ( QBC) AB 2 BC2
   
Area ( RAC) Area ( RAC) AC2 AC2
Area ( PAB) Area ( QBC) AB 2  BC2
  
Area ( RAC) Area ( RAC) AC2

Area ( PAB) Area ( QBC) AC  ABC is a right angled triangle 


   1  2
Area ( RAC) Area ( RAC) AC2  with B  90  AC  AB  BC 
2 2

Area ( PAB)  Area ( QBC)


 1
Area ( RAC)
 Area  PAB   Area  QBC   Area  ARC 

Ex.115 In figure, ABCD is a trapezium in which AB || DC and AB = 2DC. Determine the ratio of
the areas of ΔAOB and ΔCOD .
Sol. In triangle AOB and COD, we have
AOB  COD [Vertically opposite angles]
and, OAB  OCD [Alternate angles]

So, by AA-criterion of similarity, we have AOB ~ COD


Area ( AOB) AB2
 
Area ( COD) DC2
Area ( AOB) (2DC)2 4
  
Area ( COD) (DC)2 1
Hence, Area  AOB  : Area  COD  = 4 : 1.

FEAT Educational Ventures Pvt. Ltd.


H.O. 83, Heera Panna Mall, Near Jal-Vayu Vihar, Hiranandani Garden, Powai, Mumbai - 76 84
Call: +91-22-3226-8201 | SMS: +91-842-482-1664 | www.edventure.in | info@edventure.in
Foundation – IX Mathematics Geometry

Ex.116 D, E, F are the mid-point of the sides BC, CA and AB respectively of a ΔABC .
Determine the ratio of the areas of ΔDEF and ΔABC .
Sol. Since D and E are the mid-points of the sides BC and AB respectively of ABC . Therefore,
DE || BA  DE || FA ----- (I)

Since D and F are mid-points of the sides BC and AB respectively of ABC , Therefore,
DF || CA  DF || AE
From (I), and (II), we conclude that AFDE is a parallelogram.
Similarly, BDEF is a parallelogram.
Now, in DEF and ABC , we have
FDE  A Opposite angles of parallelogram AFDE 
and, DEF  B Opposite angles of parallelogram BDEF 
So, by AA-similarity criterion, we have
DEF ~ ABC
Area ( DEF) DE2 (1/ 2AB)2 1
   
Area ( ABC) AB2 AB2 4
 1 
 DE  2 AB
 
Hence, Area  DEF  : Area  ABC   1: 4 .

Ex.117 D and E are points on the sides AB and AC respectively of a ΔABC such that DE ||BC
BD
and divides ΔABC into two parts, equal in area. Find .
AB
Sol. We have,
Area  ADE  = Area  trapezium BCED   Area  ADE  + Area  ADE 
= Area  trapezium BCED  + Area  ADE   2Area  ADE  = Area  ABC 
In ADE and ABC , we have
ADE  B  DE || BC  ADE  B Corresponding angles
and, A  A [Common]
 ABC ~ ABC

Area ( ADE) AD2


 
Area ( ABC) AB2
Area ( ADE) AD2
 
2 Area ( ADE) AB2

FEAT Educational Ventures Pvt. Ltd.


H.O. 83, Heera Panna Mall, Near Jal-Vayu Vihar, Hiranandani Garden, Powai, Mumbai - 76 85
Call: +91-22-3226-8201 | SMS: +91-842-482-1664 | www.edventure.in | info@edventure.in
Foundation – IX Mathematics Geometry

2
1  AD  AD 1
   
2  AB  AB 2
 AB  2 AD
 AB  2  AB  BD 

  
2  1 AB  2 BD

BD 2 1 2 2
  
AB 2 2

Ex.118 Two isosceles triangles have equal vertical angles and their areas are in the ratio
16 : 25. Find the ratio of their corresponding heights.
Sol. Let ABC and DEF be the given triangles such that AB = AC and DE = DF, A  D .
Area ( ABC) 16
and,  ------ (I)
Area ( DEF) 25
Draw AL  BC and DM  EF.
Now, AB = AC, DE = DF
AB DE
  1 and 1
AC DF

AB DE AB AC
   
AC DF DE DF
Thus, in triangles ABC and DEF, we have
AB AC
 and A  D [Given]
DE DF
So, by SAS-similarity criterion, we have
ABC ~ DEF
Area ( ABC) AL2
 
Area ( DEF) DM2
16 AL2
  Using I
25 DM2
AL 4
   AL: DM  4 : 5
DM 5

FEAT Educational Ventures Pvt. Ltd.


H.O. 83, Heera Panna Mall, Near Jal-Vayu Vihar, Hiranandani Garden, Powai, Mumbai - 76 86
Call: +91-22-3226-8201 | SMS: +91-842-482-1664 | www.edventure.in | info@edventure.in
Foundation – IX Mathematics Geometry

Area (ΔDEF)
Ex.119 In figure, DE || BC and AD : DB = 5 : 4. Find .
Area (ΔCFB)
Sol. In ABC , we have
DE || BC
 ADE  ABC and AED  ACB Corresponding angles
Thus, in triangles ADE and ABC, we have
A  A Common
ADE  ABC
and, AED  ACB
 AED ABC By AAA similarity 
AD DE
 
AB BC

We have,
AD 5

DB 4
DB 4 DB 4
    1  1
AD 5 AD 5
DB  AD 9
 
AD 5
AB 9 AD 5
   
AD 5 AB 9
DE 5
 
BC 9
In DFE and CFB , we have
1  3  Alternate interior angles
2  4  Vertically opposite angles 
Therefore, by AA-similarity criterion, we have
DFE CFB
Area ( DFE) DE2
 
Area (CFB) BC2
2
Area ( DFE)  5  25
    Using I
Area (CFB)  9  81

FEAT Educational Ventures Pvt. Ltd.


H.O. 83, Heera Panna Mall, Near Jal-Vayu Vihar, Hiranandani Garden, Powai, Mumbai - 76 87
Call: +91-22-3226-8201 | SMS: +91-842-482-1664 | www.edventure.in | info@edventure.in
Foundation – IX Mathematics Geometry

Pythagoras theorem

Theorem 1:
 In a right angled triangle, the square of the hypotenuse is equal to the sum of the squares of
the other two sides.

 Given: A right-angled triangle ABC in which B  90 .

 To Prove: (Hypotenuse)2 = (Base)2 + (Perpendicular)2. i.e., AC2  AB2  BC2

 Construction: From B draw BD  AC.

 Proof:
In triangle ADB and ABC, we have
ADB  ABC [Each equal to 90º]
and, A  A [Common]
So, by AA-similarity criterion, we have
ADB ~ ABC
AD AB
   [ In similar triangles corresponding sides are proportional]
AB AC
 AB2  AD  AC ----- (I)
In triangles BDC and ABC, we have
CDB  ABC [Each equal to 90º]
and, C  C [Common]
So, by AA-similarity criterion, we have
BDC ~ ABC
DC BC
  [ In similar triangles corresponding sides are proportional]
BC AC
 BC2  AC  DC ----- (II)
Adding equation (I) and (II), we get
AB2  BC2  AD  AC  AC  DC
 AB2  BC2  AC  AD  DC 

 AB2  BC2  AC  AC
 AB2  BC2  AC2
Hence, AC2  AB2  BC2
The converse of the above theorem is also true as proved below.

FEAT Educational Ventures Pvt. Ltd.


H.O. 83, Heera Panna Mall, Near Jal-Vayu Vihar, Hiranandani Garden, Powai, Mumbai - 76 88
Call: +91-22-3226-8201 | SMS: +91-842-482-1664 | www.edventure.in | info@edventure.in
Foundation – IX Mathematics Geometry

Theorem 2: (Converse of Pythagoras Theorem).


 In a triangle, if the square of one side is equal to the sum of the squares of the other two
sides, then the angle opposite to the side is a right angle.

 Given: A triangle ABC such that AC2  AB2  BC2

 Construction: Construct a triangle DEF such that DE = AB, EF = BC and E  90 ,

 Proof:
In order to prove that B  90 , it is sufficient to show that ABC ~ DEF . For this we
proceed as follows:
Since DEF is a right angled triangle with right angle at E. Therefore, by Pythagoras
theorem, we have
DF2  DE2  EF2
 DF2  AB2  BC2 [ DE = AB and EF = BC (By construction)]
 DF2  AC2  AB2  BC2  AC2  Given
 
  DF = AC ----- (I)
Thus, in ABC and DEF , we have
AB = DE, BC = EF [By construction]
and, AC = DF [From equation (I)]
 ABC DEF
 B  E  90
Hence, ABC is a right triangle right angled at B.

Ex.120 In Fig., ΔABC is an obtuse triangle, obtuse angled at B. If AD  CB, prove that
AC2 = AB2 + BC2 + 2BC  BD
Sol. Given: An obtuse triangle ABC, obtuse-angled at B and AD is perpendicular to CB
produced.
To Prove: AC2  AB2  BC2  2BC  BD
Proof: Since ADB is a right triangle right angled at D. Therefore, by Pythagoras theorem,
we have
AB2  AD2  DB2 ----- (I)

Again ADC is a right triangle right angled at D.


Therefore, by Pythagoras theorem, we have
AC2  AD2  DC2
AC2  AD2  DB  BC
2

FEAT Educational Ventures Pvt. Ltd.


H.O. 83, Heera Panna Mall, Near Jal-Vayu Vihar, Hiranandani Garden, Powai, Mumbai - 76 89
Call: +91-22-3226-8201 | SMS: +91-842-482-1664 | www.edventure.in | info@edventure.in
Foundation – IX Mathematics Geometry

 AC2  AD2  DB2  BC2  2BC · BD


 AC2  AB2  BC2  2BC · BD [Using (I)]
Hence, AC  AB  BC  2BC · BD
2 2 2

Ex.121 In figure, B of ABC is an acute angle and AD  BC, prove that


AC2 = AB2 + BC2 – 2BC × BD
Sol. Given: A ABC in which B is an acute angle and AD  BC .
To Prove: AC2  AB2  BC2 – 2BC  BD .
Proof: Since ADB is a right triangle right-angled at D. So, by Pythagoras theorem, we
have AB2  AD2  BD2 ----- (I)
Again ADC is a right triangle right angled at D

So, by Pythagoras theorem, we have


AC2  AD2  DC2
AC2  AD2  BC  BD
2

 
AC2  AD2  BC2  BD2 – 2BC · BD 
  
AC2  AD2  BD2  BC2 – 2BC · BD

 AC2  AB2  BC2 – 2BC · BD Using I


Hence, AC2  AB2  BC2 – 2BC · BD

Ex.122 Prove that in any triangle, the sum of the squares of any two sides is equal to twice
the square of half of the third side together with twice the square of the median which
bisects the third side.
Sol. Given: A ABC in which AD is a median.
2
2 2 1
2

To Prove: AB  AC  2 AD  2  BC 
2


or, 
AB2  AC2  2 AD2  BD2 
Construction: Draw AE  BC .

Proof: Since AED  90 . Therefore, in ADE , we have


ADE < 90  ADB  90
Thus, ADB is an obtuse-angled triangle and ADC is an acute angled triangle.
ABD is obtuse-angled at D and AE  BD produced. Therefore, by theorem 1, we have
AB2  AD2  BD2  2BD  DE ------- (I)

FEAT Educational Ventures Pvt. Ltd.


H.O. 83, Heera Panna Mall, Near Jal-Vayu Vihar, Hiranandani Garden, Powai, Mumbai - 76 90
Call: +91-22-3226-8201 | SMS: +91-842-482-1664 | www.edventure.in | info@edventure.in
Foundation – IX Mathematics Geometry

ACD is acute-angled at D and AE  CD. Therefore, by theorem 2, we have


AC2  AD2  DC2 – 2DC  DE
 AC2  AD2  BD2 – 2BD  DE  CD  BD ------- (II)
Adding equations (I) and (II), we get
AB2  AC2  2 AD2  BD2  
 2  BC 2 
 AB  AC  2  AD  
2 2
 
  2  
2
1 
 AB  AC  2AD  2  BC 
2 2 2

2 
 AB2  AC2  2 AD2  BD2  

Ex.123 Prove that three times the sum of the squares of the sides of a triangle is equal to four
times the sum of the squares of the medians of the triangle.
Sol. Given: A ABC in which AD, BE and CF are three medians.

To Prove: 3 AB2  BC2  CA 2  4 AD2  BE2  CF2   
Proof: Since in any triangle, the sum of the squares of any two sides is equal to twice the
square of half of the third side together with twice the square of the median bisecting it.
Therefore, taking AD as the median bisecting side BC, We have

AB2  AC2  2 AD2  BD2  


 2  BC 2 
 AB  AC  2  AD  
2 2
 
  2  
  BC  
2
 AB2  AC2  2  AD2    
  4  

 
2 AB2 + AC2  4AD2 + BC2   ------- (I)
Similarly, by taking BE and CF respectively as the medians, we get
  
2 AB2  BC2  4BE2  AC2  ------- (II)

and, 2  AC 2
 BC    4CF
2 2
 AB2  ------- (III)
Adding (I), (II) and (III), we get
    
4 AB2  BC2  AC2  4 AD2  BE2  CF2  BC2  AC2  AB2 
 3  AB  BC  AC   4  AD  BE  CF 
2 2 2 2 2 2

Hence, 3  AB  BC  AC   4  AD  BE  CF 
2 2 2 2 2 2

FEAT Educational Ventures Pvt. Ltd.


H.O. 83, Heera Panna Mall, Near Jal-Vayu Vihar, Hiranandani Garden, Powai, Mumbai - 76 91
Call: +91-22-3226-8201 | SMS: +91-842-482-1664 | www.edventure.in | info@edventure.in
Foundation – IX Mathematics Geometry

Ex.124 A right triangle has hypotenuse of length p cm and one side of length q cm.
If p – q = 1, find the length of the third side of the triangle.
Sol. Let the third side be x cm. Then, by Pythagoras theorem, we have
p2 2  q2  x 2
 x 2  p2 – q2  p  q  p  q   p  q  p  q  1

 x  p  q  2q  1  p  q  1  p  q  1

Hence, the length of the third side is 2q  1 cm.

Ex.125 A man goes 10 m due east and then 24 m due north. Find the distance from the
starting point.
Sol. Let the initial position of the man be O and his final position be B. Since the man goes 10 m
due east and then 24 m due north. Therefore, AOB is a right triangle right-angled at
A such that OA = 10 m and AB = 24 m.

By Pythagoras theorem, we have


OB2  OA 2  AB2
 OB2  102  242  100  576  676
 OB = 676 = 26 m
Hence, the man is at a distance of 26 m from the starting point.

Ex.126 The hypotenuse of a right triangle is 6 m more than the twice of the shortest side. If
the third side is 2 m less than the hypotenuse, find the sides of the triangle.
Sol. Let the shortest side be x metres in length.
Then, Hypotenuse   2x  6  m and,
Third side   2x  4  m.
By Pythagoras theorem, we have
 2x + 6 2  x2   2x  4 
2

 4x2  24x  36  x2  4x 2  16x  16


 x 2 – 8x – 20  0  x  10  x  2  0
 x  10 or, x– 2
 x = 10  x cannot be negative 
Hence, the sides of the triangle are 10 m, 26 m and 24 m.

FEAT Educational Ventures Pvt. Ltd.


H.O. 83, Heera Panna Mall, Near Jal-Vayu Vihar, Hiranandani Garden, Powai, Mumbai - 76 92
Call: +91-22-3226-8201 | SMS: +91-842-482-1664 | www.edventure.in | info@edventure.in
Foundation – IX Mathematics Geometry

Ex.127 P and Q are the mid-points of the sides CA and CB respectively of a ABC, right
angled at C. Prove that :
(I) 4AQ2  4AC2  BC2 (II) 4BP2  4BC2  AC2  
(III) 4AQ2  BP2  5AB2
Sol. (I) Since AQC is a right triangle right-angled at C.
 AQ2  AC2  QC2
 4AQ2  4AC2  4QC2 [Multiplying both sides by 4]
4AQ  4AC   2QC
2
 2 2

 4AQ2  4AC2  BC2  BC  2QC


(II) Since BPC is a right triangle right-angled at C.

 BP2  BC2  CP2


 4BP2  4BC2  4CP2 [Multiplying both sides by 4]
4BP2  4BC2   2CP
2

 4BP2  4BC2  AC2  AC  2CP 
(III) From (I) and (II), we have
4AQ2  4AC2  BC2 and, 4BC2  4BC2  AC2
 
4AQ2  4BP2  4AC2  BC2  4BC2  AC2   
   
4 AQ2  BP2  5 AC2  BC 
2

 4  AQ 2
 BP   5 AB
2 2
In ABC, we have AB2  AC2  BC2 
 

Ex.128 In Figure, ABC is a right triangle right-angled at B. AD and CE are the two medians
3 5
drawn from A and C respectively. If AC = 5cm and AD = cm, find the length
2
of CE.
Sol. Since ABD is a right triangle right-angled at B. Therefore,
AD2  AB2  BD2
2
 BC 
 AD2  AB2     BD  DC
 2 
1
 AD2  AB2  · BC2 ------- (I)
4
Again, BCE is a right triangle right angled at B.

FEAT Educational Ventures Pvt. Ltd.


H.O. 83, Heera Panna Mall, Near Jal-Vayu Vihar, Hiranandani Garden, Powai, Mumbai - 76 93
Call: +91-22-3226-8201 | SMS: +91-842-482-1664 | www.edventure.in | info@edventure.in
Foundation – IX Mathematics Geometry

 CE2  BC2  BE2


2
 AB 
 CE  BC  
2 2
  BE  EA 
 2 
1
 CE2  BC2  · AB2 ------- (II)
4
Adding (I) and (II), we get
1 1
AD2  CE2  AB2  BC2  BC2  AB2
4 4
 AD2  CE2 
5
4

AB2  BC2 
5 
 AD2  CE2  AC2  ABC is right triangle  AC2  AB2  BC2 
4
2
3 5  5
    CE   25
2

 2  4
125 45
 CE2    20
4 4
 CE = 20 cm = 2 5 cm

Ex.129 ABC is a right triangle right-angled at B. Let D and E be any points on AB and BC
respectively. Prove that AE 2 + CD2 = AC 2 + DE 2 .
Sol. Since ABE is right triangle, right-angled at B.
 AE2  AB2  BE2 ------- (I)
Again, DBC is right triangle right-angled at B.
 CD2  BD2  BC2 ------- (II)

Adding (I) and (II), we get



AE2  CD2  AB2  BE2  BD2  BC2  
 AE2  CD2   AB 2
 BC   BE
2 2
 BD  2

Using Pythagoras theorem in ABC and DBE ,


we have
AC2  AB2  BC2 and DE2  BE2  BD2
 AE2  CD2  AC2  DE2

FEAT Educational Ventures Pvt. Ltd.


H.O. 83, Heera Panna Mall, Near Jal-Vayu Vihar, Hiranandani Garden, Powai, Mumbai - 76 94
Call: +91-22-3226-8201 | SMS: +91-842-482-1664 | www.edventure.in | info@edventure.in
Foundation – IX Mathematics Geometry

Hence, AE2  CD2  AC2  DE2

Ex.130 The perpendicular AD on the base BC of a ABC intersects BC at D so that DB = 3 CD.


Prove that 2AB2 = 2AC 2 + BC 2 .
Sol. We have,

DB = 3CD
 BC = BD + DC
 BC = 3 CD + CD
1
 BD = 4 CD  CD = BC
4
1 3
 CD = BC and BD = 3CD = BC ------- (I)
4 4
Since ABD is a right triangle right-angled at D.
 AB2  AD2  BD2 ------- (II)
Similarly, ACD is a right triangle right angled at D.
 AC2  AD2  CD2 ------- (III)
Subtracting equation (III) from equation (II) we get
AB2 – AC2  BD2 – CD2
2 2
3  1   1 3 
 AB2 – AC2   BC    BC  From I CD  4 BC, BD  4 BC
4  4   
9 1
 AB2 – AC2  BC2 – BC2
16 16
1
 AB2  AC2  BC2
2
  
2 AB2 – AC2  BC2

 2AB2  2AC2  BC2

Ex.131 ABC is a right triangle right-angled at C. Let BC = a, CA = b, AB = c and let p be the


length of perpendicular from C on AB, prove that
1 1 1
(I) cp = ab (II) 2 = 2 + 2
P a b
Sol. (I) Let CD  AB . Then, CD = p.

FEAT Educational Ventures Pvt. Ltd.


H.O. 83, Heera Panna Mall, Near Jal-Vayu Vihar, Hiranandani Garden, Powai, Mumbai - 76 95
Call: +91-22-3226-8201 | SMS: +91-842-482-1664 | www.edventure.in | info@edventure.in
Foundation – IX Mathematics Geometry

1
 Area of ABC = (Base  Height)
2
1 1
 Area of ABC = (AB  CD) = cp
2 2
1 1
Also, Area of ABC = (BC  AC) = ab
2 2
1 1
 cp = ab  cp = ab
2 2
(II) Since ABC is right triangle right-angled at C.
 AB2  BC2  AC2
 c 2  a2  b 2
2
 ab   ab 
   =a  b  cp  ab  c  p 
2 2

 
p  
a 2b 2 1 a2  b2
 = a2  b2  =
p2 p2 a 2b 2
1 1 1 1 1 1
 2
= 2 + 2  2
= 2 + 2
p b a p a b

Ex.132 Prove that three times the square of any side of an equilateral triangle is equal to four
times the square of the altitude.
Sol. Let ABC be an equilateral triangle and let AD  BC .
In ADB and ADC , we have
AB = AC [Given]
B  C [Each equal to 60 ]
and, ADB  ADC [Each equal to 90 ]

 ADB  ADC
 BD = DC
1
 BD = DC = BC
2
Since ADB is a right triangle right-angled at D.
 AB2  AD2  BD2
2
1 
 AB2  AD2 +  BC 
2 
BC2
 AB2  AD2 +
4
AB2
 AB2  AD2   BC  AB
4

FEAT Educational Ventures Pvt. Ltd.


H.O. 83, Heera Panna Mall, Near Jal-Vayu Vihar, Hiranandani Garden, Powai, Mumbai - 76 96
Call: +91-22-3226-8201 | SMS: +91-842-482-1664 | www.edventure.in | info@edventure.in
Foundation – IX Mathematics Geometry

3
 AB2  AD2   3AB2  4AD2
4

Ex.133 In an equilateral triangle with side a, prove that


a 3 3 2
(I) Altitude = (II) Area = a
2 4
Sol. Let ABC be an equilateral triangle the length of whose each side is a units. Draw AD  BC .
Then, D is the mid-point of BC.
1 a
 AB = a, BD = BC =
2 2
 Since ABD is a right triangle right-angled at D.
 AB2  AD2  BD2
2
a
 a2  AD2   
2
a2 3a2
 AD2  a2 – 
4 4
3
  AD = a
2
3
 Altitude a
2

Now, Area of ABC = (1/2) (Base  Height)


1
 Area of ABC = (BC  AD)
2
1 3 3 2
 Area of ABC = × a a
2 2 4

Ex.134 ABC is a triangle in which AB = AC and D is any point in BC. Prove that
AB2 – AD2 = BD · CD.
Sol. Draw AE  BC

In AEB and AEC , we have


AB = AC,
AE = AE [Common]
and, B  C [ AB = AC]

FEAT Educational Ventures Pvt. Ltd.


H.O. 83, Heera Panna Mall, Near Jal-Vayu Vihar, Hiranandani Garden, Powai, Mumbai - 76 97
Call: +91-22-3226-8201 | SMS: +91-842-482-1664 | www.edventure.in | info@edventure.in
Foundation – IX Mathematics Geometry

 AEB  AEC
 BE = CE
Since AED and ABE are right triangles right-angled at E. Therefore,
AD2  AE2  DE2 and AB2  AE2  BE2
 AB2 – AD2  BE2 – DE2
 AB2 – AD2  BE  DE  BE – DE 
 AB2 – AD2   CE  DE  BE – DE  [ BE = CE]
 AB2 – AD2  CD · BD
Hence, AB2 – AD2  BD · CD

Ex.135 From a point O in the interior of a ΔABC , perpendicular OD, OE and OF are drawn to
the sides BC, CA and AB respectively. Prove that:
(I) AF 2 + BD2 + CE 2 = OA2 + OB2 + OC2 – OD2 – OE 2 – OF 2
(II) AF 2 + BD2 + CE 2 = AE 2 + CD2 + BF 2
Sol. Let O be a point in the interior of ABC and let OD  BC, OE  CA and OF  AB .
(I) In right triangles OFA, ODB and OEC , we have
OA 2  AF2  OF2
OB2  BD2  OD2
and, OC2  CE2  OE2
Adding all these results, we get
OA 2  OB2  OC2  AF2  BD2  CE2  OF2  OD2  OE2
 AF2  BD2  CE2  OA 2  OB2  OC2 – OD2 –­ OE2 – OF2
(II) In right triangles ODB and ODC , we have

OB2  OD2  BD2


and, OC2  OD2  CD2
 
OB2  OC2  OD2  BD2  OD2  CD2  
 OB2 – OC2  BD2 – CD2 ------- (I)
Similarity, we have
OC2 – OA 2  CE2 – AE2 ------- (II)
and, OA – OB  AF – BF
2 2 2 2
------- (III)
Adding (I), (II) and (III), we get
OB 2
   
– OC2  OC2 – OA 2  OA 2 – OB2 
 BD 2
   
– CD2  CE2 – AE2  AF2 – BF2 
 BD 2
 
 CE2  AF2  AE2  CD2  BF2  0 
FEAT Educational Ventures Pvt. Ltd.
H.O. 83, Heera Panna Mall, Near Jal-Vayu Vihar, Hiranandani Garden, Powai, Mumbai - 76 98
Call: +91-22-3226-8201 | SMS: +91-842-482-1664 | www.edventure.in | info@edventure.in
Foundation – IX Mathematics Geometry

 AF2  BD2  CE2  AE2  BF2  CD2

Ex.136 A point O in the interior of a rectangle ABCD is joined with each of the vertices A, B, C
and D. Prove that OB2 + OD2 = OC2 + OA2
Sol. Let ABCD be the given rectangle and let O be a point within it. Join OA, OB, OC and OD.
Through O, draw EOF || AB . Then, ABFE is a rectangle.

In right triangles OEA and OFC , we have


OA 2  OE2  AE2 and OC2  OF2  CF2
 
OA 2  OC2  OE2  AE2  OF2  CF2   
 OA 2  OC2  OE2  OF2  AE2  CF2 ------- (I)
Now, in right triangles OFB and ODE , we have
OB2  OF2  FB2 and OD2  OE2  DE2
 
OB2  OD2  OF2  FB2  OE2  DE2   
 OB2  OD2  OE2  OF2  DE2  BF2
 OB2  OD2  OE2  OF2  CF2  AE2 [ DE = CF and AE = BF] ------- (II)
From (I) and (II), we get
OA 2  OC2  OB2  OD2

Ex.137 ABCD is a rhombus. Prove that AB2 + BC2 + CD2 + DA2 = AC2 + BD2
Sol. Let the diagonals AC and BD of rhombus ABCD intersect at O.
Since the diagonals of a rhombus bisect each other at right angles.
 AOB  BOC  COD  DOA  90
and AO  CO, BO  OD .
Since AOB is a right triangle right-angled at O.

 AB2  OA 2  OB2
2 2
1  1 
 AB   AC    BD 
2
 OA  OC and OB  OD 
2  2 
 4AB2  AC2  BD2 ------- (I)
Similarly, we have
4BC2  AC2  BD2 ------- (II)
4CD  AC  BD
2 2 2
------- (III)

FEAT Educational Ventures Pvt. Ltd.


H.O. 83, Heera Panna Mall, Near Jal-Vayu Vihar, Hiranandani Garden, Powai, Mumbai - 76 99
Call: +91-22-3226-8201 | SMS: +91-842-482-1664 | www.edventure.in | info@edventure.in
Foundation – IX Mathematics Geometry

and, 4AD2  AC2  BD2 ------- (IV )


Adding all these results, we get
  
4 AB2  BC2  AD2  4 AC2  BD2 
 AB2  BC2  CD2  DA 2  AC2  BD2

Ex.138 In a triangle ABC, AC > AB, D is the mid-point of BC and  . Prove that:
1
(I) AC 2 = AD2 + BC · DE + BC 2
4
1
(II) AB2 = AD2 – BC · DE + BC 2
4
1
(III) AB2 + AC 2 = 2AD2 + BC 2
2
Sol. We have, AED  90 ,
 ADE < 90 and ADC  90 ,
i.e., ADE is acute and ADC is obtuse.

(I) In ADC, ADC is an obtuse angle.


 AC2  AD2  DC2  2DC · DE
2
1  1
 AC2  AD2   BC   2 · BC · DE
2  2
1
 AC2  AD2  BC2  BC · DE
4
1
 AC2  AD2  BC · DE  BC2
4
(II) In ABD, ADE is an acute angle.
 AB2  AD2  BD2 – 2BD · DE
2
1  1
 AB2  AD2   BC  – 2 · BC · DE
2  2
1
 AB2  AD2  BC2 – BC · DE
4
1
 AB2  AD2 – BC · DE  BC2 ------- (II)
4
(III) From (I) and (II), we get
1
AB2  AC2  2AD2  BC2
2

FEAT Educational Ventures Pvt. Ltd.


H.O. 83, Heera Panna Mall, Near Jal-Vayu Vihar, Hiranandani Garden, Powai, Mumbai - 76 100
Call: +91-22-3226-8201 | SMS: +91-842-482-1664 | www.edventure.in | info@edventure.in
Foundation – IX Mathematics Geometry

Ex.139 In an equilateral triangle ABC the side BC is trisected at D. Prove that 9AD 2 = 7AB2
Sol. Let ABC be an equilateral triangle and let D be a point on BC such that BD = BC. Draw
AE  BC . Join AD.

In AEB , and AEC , we have AB = AC,


AEB  AEC  90
and, AE = AE
So, by RHS-criterion of similarity, we have
AEB AEC
 BE = EC
Thus, we have
1 2
BD  BC, DC  BC
3 3
1
and BE  BC ------- (I)
2
Since C  60 . Therefore, ADC is an acute triangle.
 AD2  AC2  DC2 – 2DC  EC
 Using I
4 2
 AD2  AC2  BC2 – BC2
9 3
4 2
 AD2  AB2  AB2 – AB2 [ AB = BC = AC]
9 3
9AB2  4AB2  6AB2 7
 AD2   AB2
9 9
 9AD2  7AB2

Ex.140 In a ΔABC , AD  BC and AD 2 = BC × CD . Prove that ΔABC is a right triangle.


Sol. In right triangles ADB and ADC, we have
AB2  AD2  BD2 ------- (I)

and, AC2  AD2  DC2 ------- (II)


Adding (I) and (II), we get
AB2  AC2  2AD2  BD2  DC2

FEAT Educational Ventures Pvt. Ltd.


H.O. 83, Heera Panna Mall, Near Jal-Vayu Vihar, Hiranandani Garden, Powai, Mumbai - 76 101
Call: +91-22-3226-8201 | SMS: +91-842-482-1664 | www.edventure.in | info@edventure.in
Foundation – IX Mathematics Geometry

 AB2  AC2  2BD  CD  BD2  DC2 


 AD2  BD  CD Given
 AB2  AC2  BD  CD  2  BC2
Thus, in ABC , we have
AB2  AC2  BC2
Hence, ABC , is a right triangle right-angled at A.

Ex.141 In fig, D and E trisect BC. Prove that 8 AE 2 = 3 AC 2 + 5 AD 2


Sol. Since D and E are the points of trisection of BC. Therefore,
BD = DE = CE.
Let BD = DE = CE = x.
Then, BE = 2x and BC = 3x.
In right triangles ABD, ABE and ABC, we have

 AD2  AB2  BD2


 AD2  AB2  x 2 ------- (I)
AE2  AB2  BE2
 AE2  AB2  4 x 2 ------- (II)
and, AC  AB  BC
2 2 2

 AC2  AB2  9 x 2 ------- (III)


Now,   
8 AE2 – 3 AC2 – 5 AD2  8 AB2  4x2 – 3 AB2  9x 2 – AB2  x 2   
 8 AE2 – 3 AC2 – 5 AD2  0
 8 AE2  3 AC2  5 AD2

Ex.142 ABC is an isosceles triangle right-angled at B. Similar triangles ACD and ABE are
constructed on sides AC and AB. Find the ratio between the areas of ΔABE and
ΔACD .
Sol. Let AB = BC = x.
It is given that ABC is right-angled at B.
 AC2  AB2  BC2
 AC2  x 2  x2
 AC  2 x

FEAT Educational Ventures Pvt. Ltd.


H.O. 83, Heera Panna Mall, Near Jal-Vayu Vihar, Hiranandani Garden, Powai, Mumbai - 76 102
Call: +91-22-3226-8201 | SMS: +91-842-482-1664 | www.edventure.in | info@edventure.in
Foundation – IX Mathematics Geometry

It is given that
ABE ACD
Area(  ABE) AB2
 
Area(  ACD) AC2
Area(  ABE) x2
 
Area(  ACD)
 
2
2x
Area(  ABE) 1
 
Area(  ACD) 2

Ex.143P and Q are points on sides CA and CB respectively of ABC right angled at C. Prove
that AQ2 + BP 2 = AB2 + PQ 2
Sol. In right-angled triangles ACQ and PCB, we have

AQ2  AC2 + CQ2 and PB2  PC2  CB2

   
AQ2  BP2  AC2  CQ2  PC2  CB2 
 AQ2  BP2   AC 2
 BC   PC 2 2
 QC 
2

 AQ2  BP2  AB2  PQ2


[By Pythagoras theorem, we have AC2  BC2  AB2 and PC2  QC2  PQ2 ]

Ex.144 ABC is a right-angled triangle right angled at A. A circle is inscribed in it the lengths
of the two sides containing the right angle are 6 cm and 8 cm. Find the radius of the
circle.
Sol. Using Pythagoras theorem in BAC , we have

BC2  AB2  AC2


 BC2  62  82  100   BC = 10 cm
Now, Area of ABC = Area of OAB + Area of  OBC + Area of OCA

FEAT Educational Ventures Pvt. Ltd.


H.O. 83, Heera Panna Mall, Near Jal-Vayu Vihar, Hiranandani Garden, Powai, Mumbai - 76 103
Call: +91-22-3226-8201 | SMS: +91-842-482-1664 | www.edventure.in | info@edventure.in
Foundation – IX Mathematics Geometry

1 1 1 1
 AB  AC  AB  r  BC  r  CA  r
2 2 2 2
1 1 1 1
  6  8   6  r   10  r   8  r 
2 2 2 2
 48 = 24 r   r = 2 cm

Ex.145 ABC is an isosceles triangle with AC = BC. If AB2 = 2AC2 , prove that ΔABC is right
triangle.
Sol. We have, AC = BC and AB2  2AC2
Now, AB2  2 AC2
 AB2  AC2  AC2
 AB2  AC2  BC2  AC  BC Given
 ABC is a right triangle right angled at C.

Ex.146 In a right triangle ABC right-angled at C, P and Q are the points on the sides CA and
CB respectively, which divide these sides in the ratio 2 : 1. Prove that
(I) 9 AQ 2 = 9 AC 2 + 4 BC 2 (II) 9 BP 2 = 9 BC 2 + 4 AC 2  
(III) 9 AQ2 + BP 2 = 13 AB2
Sol. It is given that P divides CA in the ratio 2 : 1. Therefore,
2
CP = AC ------- (I)
3
Also, Q divides CB in the ratio 2 : 1.
2
 QC = BC ------- (II)
3

(I) Applying Pythagoras theorem in right-angled triangle ACQ, we have


AQ2  QC2  AC2
4
 AQ2  BC2  AC2 Using II
9
 9 AQ2  4BC2  9 AC2 ------- (III)
(II) Applying Pythagoras theorem in right triangle BCP, we have
BP2  BC2  CP2
4
 BP2  BC2  AC2 Using I
9
 9BP2  9BC2  4 AC2 ------- (IV)
Adding (III) and (IV), we get
  
9 AQ2  BP2  13 BC2  AC2 

FEAT Educational Ventures Pvt. Ltd.


H.O. 83, Heera Panna Mall, Near Jal-Vayu Vihar, Hiranandani Garden, Powai, Mumbai - 76 104
Call: +91-22-3226-8201 | SMS: +91-842-482-1664 | www.edventure.in | info@edventure.in
Foundation – IX Mathematics Geometry

  
9 AQ2  BP2  13 AB2 
 BC2  AC2  AB2 

Ex.147 If A be the area of a right triangle and b one of the sides containing the right angle,
2Ab
prove that the length of the altitude on the hypotenuse is .
b 4 + 4A2
Sol. Let PQR be a right triangle right-angled at Q such that QR = b and A = Area of PQR
Draw QN perpendicular to PR.
We have, A = Area of PQR
1 1
 A   QR  PQ   A  b  PQ 
2 2
2A
 PQ  ------- (I)
b

Now, in ’s PNQ and PQR, we have


   PNQ   PQR Each equal to 90
and, QPN  QPR [Common]
So, by AA-criterion of similarity, we have
PNQ PQR
PQ NQ
  ------- (II)
PR QR
Applying Pythagoras theorem in PQR , we have
PQ2  QR2  PR2
4A 2
   b2  PR2
b2
4A 2  b 4 4A 2  b 4
  PR  
b2 b
From (I) and (II), we have
2A NQ

b  PR b
2A 2Ab
 NQ   NQ 
PR 4A 2  b 4

FEAT Educational Ventures Pvt. Ltd.


H.O. 83, Heera Panna Mall, Near Jal-Vayu Vihar, Hiranandani Garden, Powai, Mumbai - 76 105
Call: +91-22-3226-8201 | SMS: +91-842-482-1664 | www.edventure.in | info@edventure.in
Foundation – IX Mathematics Geometry

Important points to be remembered


 Two figures having the same shape but not necessarily the same size are called similar
figures.

 All congruent figures are similar but the converse is not true.

 Two polygons having the same number of sides are similar, if


 Their corresponding angles are equal and
 Their corresponding sides are proportional (i.e., in the same ratio)

 If a line is drawn parallel to one side of a triangle to intersect the other two sides in distinct
points, then the other two sides are divided in the same ratio.

 If a line divides any two sides of a triangle in the same ratio, then the line is parallel to the
third side of the triangle.

 The internal bisector of an angle of a triangle divides the opposite side internally in the ratio
of the sides containing the angle.

 If a line through one vertex of a triangle divides the opposite side in the ratio of other two
sides, then the line bisects the angle at the vertex.

 The external bisector of an angle of a triangle divides the opposite side externally in the ratio
of the sides containing the angle.

 The line drawn from the mid-point of two sides of a triangle is parallel of another side bisects
the third side.

 The line joining the mid-points of two sides of a triangle is parallel to the third side.

 The diagonals of a trapezium divide each other proportionally.

 If diagonals of a quadrilateral divide each other proportionally, then it is a trapezium.

 Any line parallel to the parallel sides of a trapezium divides the non-parallel sides
proportionally.

 If three or more parallel lines are intersected by two transversals, then the intercepts made
by them on the transversals are proportional.

 AAA similarity criterion: If in two triangles, corresponding angles are equal, then the triangles
are similar.

 AA Similarity criterion: If in two triangles, two angles of one triangle are respectively equal
the two angles of the other triangle, and then the two triangles are similar.

 SSS Similarity criterion: If in two triangles, corresponding sides are in the same ratio, then
the two triangles are similar.

FEAT Educational Ventures Pvt. Ltd.


H.O. 83, Heera Panna Mall, Near Jal-Vayu Vihar, Hiranandani Garden, Powai, Mumbai - 76 106
Call: +91-22-3226-8201 | SMS: +91-842-482-1664 | www.edventure.in | info@edventure.in
Foundation – IX Mathematics Geometry

 If one angle of a triangle is equal to one angle of another triangle and the sides including
these angles are in the same ratio, then the triangles are similar.

 If two triangles are equiangular, then


 The ratio of the corresponding sides is same as the ratio of corresponding median.
 The ratio of the corresponding sides is same as the ratio of the corresponding angle
bisector segments.
 The ratio of the corresponding sides is same as the ratio of the corresponding altitudes.

 If one angle of a triangle is equal to one angle of another triangle and the bisectors of these
equal angles divide the opposite side in the same ratio, then the triangles are similar.

 If two sides and a median bisecting one of these sides of a triangle are respectively
proportional to the two sides and the corresponding median of another triangle, then the
triangles are similar.

 If two sides and a median bisecting the third side of a triangle are respectively proportional to
the two sides and the corresponding median of another triangle, when the triangles are
similar.

 The ratio of the areas of two similar triangles is equal to the ratio of
 The squares of any two corresponding sides
 The squares of the corresponding altitudes.
 The squares of the corresponding medians.
 The squares of the corresponding angle bisector segments

 If the areas of two similar triangles are equal, then the triangles are congruent i.e., equal and
similar triangles congruent.

 If a perpendicular is drawn from the vertex of the right angle of a right triangle to the
hypotenuse, then the triangles on both sides of the perpendicular are similar to the whole
triangle and also to each other.

 Pythagoras Theorem: In a right triangle, the square of the hypotenuse is equal to the sum of
the squares of the other two sides.

 Converse of Pythagoras Theorem: If in a triangle, square of one side is equal to the sum of
the squares of the other two sides, then the angle opposite to first side is a right angle.

 In any triangle, the sum of the squares of any two sides is equal to twice the square of half of
the third side together with the twice of the square of the median which bisects the third side.

 Three times the sum of the squares of the sides of a triangle is equal to four times the sum
of the squares of the medians of the triangle.

 Three times the square of any side of an equilateral triangle is equal to four times the square
of the altitude.

FEAT Educational Ventures Pvt. Ltd.


H.O. 83, Heera Panna Mall, Near Jal-Vayu Vihar, Hiranandani Garden, Powai, Mumbai - 76 107
Call: +91-22-3226-8201 | SMS: +91-842-482-1664 | www.edventure.in | info@edventure.in
Foundation – IX Mathematics Geometry

EXERCISE – I
Very short answer type questions

1. If the ratio of three angles of a triangle is 1 : 2 : 3, find the angles.

2. In the fig.  || m and n is transversal. PO and QO are angle bisectors. Prove that POQ = 90º.
n

 P

O
m
Q

3. If the angles of a triangle are in the ratio 2 : 3 : 4, determine the three angles.

4. The sum of two angles of a triangle is 95º and their difference is 25º. Find all the three
angles of the triangle.

5. The side BC of a triangle ABC is produced to D. The bisector of the A meets BC in L.


Prove that ABC + ACD = 2 ALC.

6. The sides BC, CA and AB of ABC, are produced In order, forming exterior angles ACD,
BAE and CBF. Show that ACD + BAE + CBF = 360º.

7. Sides BC, CA and BA of the ABC are produced to D, E, F, respectively. If ACD = 110°
and EAF = 130º. Find all the three angles of the triangle.

8. In the adjoining figure, find the value of, A + B + C + D + E + F.


A

D E

B C

9. In figure, DP = BQ, DPB = BQD and ADP = CBQ


D Q C

A P B

Show that ADP CBQ.

10. Prove that hypotenuse is the longest side in a right-angled-triangle.

11. Prove that the sum of the three sides of a triangle is greater than the sum of its three
medians.

FEAT Educational Ventures Pvt. Ltd.


H.O. 83, Heera Panna Mall, Near Jal-Vayu Vihar, Hiranandani Garden, Powai, Mumbai - 76 108
Call: +91-22-3226-8201 | SMS: +91-842-482-1664 | www.edventure.in | info@edventure.in
Foundation – IX Mathematics Geometry

12. In Fig. the sides BA and CA have been produced such that BA = AD and CA = AE. Prove
that segment DE || side BC.
E D

B C

13. In Fig. PA  AB, QB  AB and PA = QB. If PQ intersects AB at O , show that O is the mid-
point of AB as well as that of PQ.
Q

A O
B

14. In a ABC, D and E are points on the sides AB and AC respectively such that DE||BC.
(I) If AD = 6cm, DB = 9cm and AE = 8 cm, find AC.
AD 3
(II) If  and AC = 15 cm find AE.
DB 4
AD 2
(III) If  and AC = 18 cm, find AE
DB 3
(IV) If AD = 4, AE = 8, DB = x – 4 and EC = 3x – 19, find x.
(V) If AD = 8 cm, AB = 12 cm and AE = 12 cm, find CE.
(VI) If AD = 4 cm, DB = 4.5 cm and AE = 8 cm, find AC.
(VII) If AD = 2 cm, AB = 6 cm and AC = 9 cm, find AE.
AD 4
(VIII) If  and EC = 2.5 cm, find AE.
BD 5
(IX) If AD = x, DB = x – 2, AE = x + 2 and EC = x – 1, find the value of x.
(X) If AD = 8x – 7, DB = 5x – 3, AE = 4x – 3 and EC = (3x – 1), find the value of x.
(XI) AD = 4x – 3, AE = 8x – 7, BD = 3x – 1 and CE = 5x – 3, find the volume of x.

15. In a ABC, D and E are points on the sides AB and AC respectively. For each of the
following cases show that DE||BC:
(I) AB = 12 cm, AD = 8 cm, AE = 12 cm and AC = 18 cm.
(II) AB = 5.6 cm, AD = 1.4 cm, AC = 7.2 cm and AE = 1.8 cm
(III) AB = 10.8 cm, BD = 4.5 cm, AC = 4.8 cm and AE = 2.8 cm.
(IV) AD = 5.7 cm, BD = 9.5 cm, AE = 3.3 cm and EC = 5.5 cm

16. In a  ABC, AD is the bisector of A, meeting side BC at D.


(I) If BD = 2.5 cm, AB = 5 cm and AC = 4.2 cm, find DC.
(II) If BD = 2cm, AB = 5 cm and DC = 3 cm, find AC
(III) If AB = 3.5 cm, AC = 4.2 cm and DC = 2.8 cm, find BD.
(IV) If BC = 10 cm, AC = 14 cm and BC = 6 cm, find BD and DC.
(V) If AC = 4.2 cm, DC = 6 cm, BC = 10 cm, find AB.
(VI) If AB = 5.6, AC = 6 cm and DC = 3 cm, find BC.
(VII) If AB = 5.6 cm, BC = 6 cm and BD = 3.2 cm find AC.
(VIII) If AB = 10 cm, AC = 6cm, and BC = 12 cm, find BD and DC.

FEAT Educational Ventures Pvt. Ltd.


H.O. 83, Heera Panna Mall, Near Jal-Vayu Vihar, Hiranandani Garden, Powai, Mumbai - 76 109
Call: +91-22-3226-8201 | SMS: +91-842-482-1664 | www.edventure.in | info@edventure.in
Foundation – IX Mathematics Geometry

BD BC
17. In ABC, B = 2 C and the bisector of B intersects AC and D. Prove that  .
DA BA

18. In fig. if AB  BC and DE  AC. Prove that ABC ~ AED.

19. In fig. if P = RTS, prove that RPQ ~ RTS.

20. In fig. AD and CE are two altitudes of ABC.


Prove that
(I) AEF ~  CDF
(II) ABD ~  CBE
(III) AEF ~  ADB
(IV) FDC ~  BEC

21. In fig. if BD  AC and CE  AB,


Prove that
(I) AEC ~ ADB
CA CE
(II) 
AB DB

22. E is a point on side AD produced of a parallelogram ABCD and BE intersects CD at F. Prove


that ABE ~ CFB.

23. In fig. E is a point on side CB produced of an isosceles triangle ABC with AB = AC. If
AD  BC and EF  AC , prove that ABD ECF .

FEAT Educational Ventures Pvt. Ltd.


H.O. 83, Heera Panna Mall, Near Jal-Vayu Vihar, Hiranandani Garden, Powai, Mumbai - 76 110
Call: +91-22-3226-8201 | SMS: +91-842-482-1664 | www.edventure.in | info@edventure.in
Foundation – IX Mathematics Geometry

Short answer type questions

24. A triangle ABC is right angled at A. AL is perpendicular to BC. Prove that BAL = BCA.

25. In the Fig. PS is the bisector of the P and PT  QR, then show that
P

Q T S R
1
TPS = (Q – R)
2

26. In the Fig. AM  BC and AN is th angle bisector of A if B = 60º and C = 50º, find
MAN.
A

60° 50°
B C
M N

27. In the given figure, AM  BC and AN is the bisector of BAC. If B = 70º and C = 35º, find
MAN.
A

70° 35°
B M N C

28. In the figure find the value of xº.


A

25°
E

35° 60°
B C D

29. Prove that the sum of the three altitudes of a triangle is less than the sum of the three sides
of the triangle.

30. Prove that the medians of an equilateral triangle are equal.

31. Angles A, B, C of a triangle ABC are equal to each other. Prove that ABC is equilateral.

32. ABCD is a square, X and Y are points on sides AD and BC respectively such that AY = BX.
Prove that BY = AX and BAY = ABX.

FEAT Educational Ventures Pvt. Ltd.


H.O. 83, Heera Panna Mall, Near Jal-Vayu Vihar, Hiranandani Garden, Powai, Mumbai - 76 111
Call: +91-22-3226-8201 | SMS: +91-842-482-1664 | www.edventure.in | info@edventure.in
Foundation – IX Mathematics Geometry

Long Answer Type Questions

33. Prove that any two sides of a triangle are together greater than twice the median drawn to
the third side.

34. In the Fig. AP is the shortest line segment that can be drawn from A to line m. If PR > PQ,
prove that AR > AQ.
A

Q P R m

P Q
35. In quad. PQRS, (Fig.)
Prove that O
(I) PQ + QR + RS + SP > PR + QS
(II) PQ + QR + RS + SP < 2 (PR + QS) S R

36. In the Fig. PQ = PR and S is a point on PR, prove that RS < QS.
Q

R
P S

37. In  ABC, AC > AB and D is the point on AC such that AB = AD. Prove that CD < BC.

38. In the given Fig. T is a point on side QR of PQR and S is a point such that TR = TS. Prove
that PQ + PR > QS.
Q

S T

R P

39. In Fig. If AOC + BOD = 70º, find COD.


C D

A O B

40. In Fig. DFP, EDQ and FER are exterior angles of  DEF.
Prove that  DFP +  EDQ +  FER = 360º
Q

E
F P
R

FEAT Educational Ventures Pvt. Ltd.


H.O. 83, Heera Panna Mall, Near Jal-Vayu Vihar, Hiranandani Garden, Powai, Mumbai - 76 112
Call: +91-22-3226-8201 | SMS: +91-842-482-1664 | www.edventure.in | info@edventure.in
Foundation – IX Mathematics Geometry

41. In figure, AB = AC = AD. Prove that BCD = 90º.


D

Q
B C

42. In figure, AD = CF, AB = DE and BAC = EDF. Prove that ABC DEF.
A

D
E

B C

43. An exterior angle of a triangle is 120º. One of the interior opposite angle is 35º. Find theother
two angles.

FEAT Educational Ventures Pvt. Ltd.


H.O. 83, Heera Panna Mall, Near Jal-Vayu Vihar, Hiranandani Garden, Powai, Mumbai - 76 113
Call: +91-22-3226-8201 | SMS: +91-842-482-1664 | www.edventure.in | info@edventure.in
Foundation – IX Mathematics Geometry

EXERCISE – II
Short answer type questions

1. If the sides of a triangle are in the ratio 5 : 4 : 3, then find the respective altitudes on them ?

2. In the adjoining figure, find the measure of BAC, ACD?


A

65° 65° 30°


B C D

3. In the figure above, which of the following is true -?


(A) BC < CA < CD (B) BC < CA > CD
(C) BC = CA > CD (D) BC > CA = CD

4. In figure, if AB || CD and PQD = 135º, then find APE.


E
A P B

135º

C Q D
F

5. In the adjoining figure, if B = ACB = 65º and D = 30º, then –


A

B C D
(A) BC > CA > CD (B) AB = CA < CD
(C) BC < CA, CA > CD (D) BC > CA, CA < CD

6. In quadrilateral ABCD (figure), AB = AD and BC = CD then find the angle which is equal to
ABC.
D

A B

7. In this figure, PQ || TU || SR. Which of the following is true -?


(A) 1 + 2 = 3 + 4 S 1 R
6 4
(B) 1 + 2 = 5 + 6 T 3 O
U
(C) 1 + 3 = 2 + 4 5
2 Q
(D) All are ture P

FEAT Educational Ventures Pvt. Ltd.


H.O. 83, Heera Panna Mall, Near Jal-Vayu Vihar, Hiranandani Garden, Powai, Mumbai - 76 114
Call: +91-22-3226-8201 | SMS: +91-842-482-1664 | www.edventure.in | info@edventure.in
Foundation – IX Mathematics Geometry

8. In figure, if AB || CD, CD || EF and y : z = 4 : 5, then find x.


t

A B
x
C y D

z
E F

9. In Fig. it is given that BC = CE and 1 = 2. Prove that  GCB  DCE.


G D

C
1 2
B E
A F

10. In Fig. the line segment joining the mid-points M and N of opposite sides AB and DC of
quadrilateral ABCD is perpendicular to both these sides. Prove that the other sides of the
quadrilateral are equal.
A M B
90°

90°
D N C

11. In Fig. AC = AE, AB = AD and BAD = EAC. Prove that BC = DE.


E

B D C

12. In Fig. explain how one can find the breadth of the river without crossing it.
A

B O M

13. In a right angled triangle, one acute angle is double the other. Prove that the hypotenuse is
double the smallest side.

14. Show that the difference of any two sides of a triangle is less than the third side.

15. In Fig. TQ and TR are the bisectors of Q and R respectively.


If QPR  80 and PRT  30 , determine TQR .
P

Q R

FEAT Educational Ventures Pvt. Ltd.


H.O. 83, Heera Panna Mall, Near Jal-Vayu Vihar, Hiranandani Garden, Powai, Mumbai - 76 115
Call: +91-22-3226-8201 | SMS: +91-842-482-1664 | www.edventure.in | info@edventure.in
Foundation – IX Mathematics Geometry

16. In Fig. it is given that AB = CF, EF = BD and AEF = DBC. Prove that AFE ~ CBD.
D

A F C
B

17. In fig, AE is the bisector of the exterior CAD meeting BC produced in E. If AB  10 cm ,


AC  6 cm and BC  12 cm , find CE.

18. D, E and F are the points on sides BC, CA and AB respectively of ABC such that AD
bisects A, BE bisects B and CF bisects C. If AB = 5 cm, BC = 8 cm and CA = 4 cm,
determine AF, CE and BD.

19. (I) In fig.1, if AB || CD, find the value of x.


(II) In fig.2, if AB || CD, find the value of x.

Fig.1 Fig.2 Fig.3


(III) In fig.3, AB||CD. If OA = 3x – 19, OB = x – 4, OC = x – 3 and OD = 4, find x.

20. In a ABC, D and E are points on sides AB and AC respectively such that BD = CE. If
B  C , show that DE || BC.

AD BE
21. In fig. if  and CDE = CED, prove that  CAB is isosceles.
DC EC

22. In  ABC, D is the mid-point of BC and ED is the bisector of the ADB and EF is drawn
parallel to BC cutting AC in F. Prove that EDF is a right angle.

FEAT Educational Ventures Pvt. Ltd.


H.O. 83, Heera Panna Mall, Near Jal-Vayu Vihar, Hiranandani Garden, Powai, Mumbai - 76 116
Call: +91-22-3226-8201 | SMS: +91-842-482-1664 | www.edventure.in | info@edventure.in
Foundation – IX Mathematics Geometry

23. The bisectors of the angles B and C of a triangle ABC, meet the opposite side in D and E
respectively. If DE||BC, prove that the triangle is isosceles.

QT QR
24. In fig. if  and 1 = 2. Prove that PQS ~ TQR
PR QS

25. If CD and GH (D and H lie on AB and FE) are respectively bisectors of ACB and EGF
and ABC ~ FEG, prove that
(I) DCA ~ HGF
CD AC
(II) 
GH FG
(III) DCB ~ HGE

26. If ABC, if AD  BC and AD2 = BD × DC, prove that BAC = 90º

BD DA
27. In fig. if AD  BC and  , prove that  ABC is a right triangle.
DA DC

28. ABC is an isosceles right triangle, right angled at C. Prove that AB2 = 2 AC2.

29. In an isosceles triangle ABC, with AB = AC, BD is perpendicular from B to the side AC.
Prove that BD2 – CD2 = 2 CD · AD

30. In a  ABC, the angles at B and C are acute. If BE and CF be drawn perpendiculars on AC
and AB respectively, prove that BC2 = AB × BF + AC × CE.

31. ABC is a right triangle, right angled at C and AC  3BC . Prove that ABC = 60º

32. In a right-angled triangle if a perpendicular is drawn from the right angle to the hypotenuse,
prove that the square of the perpendicular is equal to the rectangle contained by the two
segments of the hypotenuse.

Long answer type questions

33. ABCD is a quadrilateral; P, Q, R and S are the points of trisection of sides AB, BC, CD and
DA respectively and are adjacent to A and C; prove that PQRS is a parallelogram.

34. In ABC, the bisector of B meets AC at D. A line PQ||AC meets AB, BC and BD at P, Q
and R respectively. Show that
(I) PR · BQ = QR·BP
(II) AB × CQ = BC × AP.

FEAT Educational Ventures Pvt. Ltd.


H.O. 83, Heera Panna Mall, Near Jal-Vayu Vihar, Hiranandani Garden, Powai, Mumbai - 76 117
Call: +91-22-3226-8201 | SMS: +91-842-482-1664 | www.edventure.in | info@edventure.in
Foundation – IX Mathematics Geometry

35. In fig. CD and GH are respectively the medians of  ABC and  EFG. If ABC ~ FEG.
Prove that

CD AB
(I) ADC ~ FHG (II)  (III) CDB ~ GHE
GH FE

36. In trapezium ABCD, AB||DC and DC = 2 AB. EF drawn parallel to AB cuts AD in F and BC in
3
E such that  . Diagonal DB intersects EF at G. Prove that 7 FE = 10 AB.
4

37. Through the vertex D of a parallelogram ABCD, a line is drawn to intersect the sides BA and
DA FB FC
BC produced at E and F respectively. Prove that  
AE BE CD

38. In fig. ABC is a right triangle right angled at B and D is the foot of the
perpendicular drawn from B on AC. If DM  BC and DN  AB,
Prove that
(I) DM2 = DN × MC
(II) DN2 = DM × AN

39. In fig. AD and BE are respectively perpendiculars to BC and AC.


Show that
(I)  ADC ~  BEC
(II) CA × CE = CB × CD
(III)  ABC ~  DEC
(IV) CD × AB = CA × DE

40. ABC is an isosceles triangle with AB = AC and D is a point on AC such that BC 2 = AC × CD.
Prove that BD = BC.

41. In  PQR, QM  PR and PR2 – PQ2 = QR2. Prove that QM2 = PM × MR.

42. Prove that the sum of the squares of the diagonals of a parallelogram is equal to the sum of
the squares of its sides.

FEAT Educational Ventures Pvt. Ltd.


H.O. 83, Heera Panna Mall, Near Jal-Vayu Vihar, Hiranandani Garden, Powai, Mumbai - 76 118
Call: +91-22-3226-8201 | SMS: +91-842-482-1664 | www.edventure.in | info@edventure.in
Foundation – IX Mathematics Geometry

EXERCISE – III
Single Correct Type Questions

1. For a triangle ABC, the true statement is–


(A) AC2 = AB2 + BC2 (B) AC = AB + BC
(C) AC > AB + BC (D) AC < AB + BC

2. If AD, BE and CF are the medians of a triangle ABC, then the true
statement is –
(A) AB2 + BC2 + AC2 = AD2 + BE2 + CF2
(B) 2 (AB2 + BC2 + AC2) = 3 (AD2 + BE2 + CF2)
(C) 3 (AB2 + BC2 + AC2) = 4 (AD2 + BE2 + CF2)
(D) AB2 + BC2 + AC2 = 3 (AD2 + BE2 + CF2)

3. The distance between the tops of two trees 20 m and 28 m high is 17 m. The horizontal
distance between the trees is –
(A) 11m (B) 31m (C) 15m (D) 9m

4. Triangle ABC is such that AB = 3 cm, BC = 2 cm and CA = 2.5 cm. Triangle DEF is similar to
ABC . If EF = 4 cm, then the perimeter of DEF is –
(A) 7.5 cm (B) 15 cm (C) 22.5 cm (D) 30 cm

5. In ABC , AB = 3 cm, AC = 4 cm and AD is the bisector of A . Then, BD : DC is –


(A) 9 : 16 (B) 16 : 9 (C) 3 : 4 (D) 4 : 3

6. In an equilateral triangle ABC, if AD  BC, then


(A) 2AB2  3AD2 (B) 4AB2  3AD2 (C) 3AB2  4AD2 (D) 3AB2  2AD2

7. ABC is a triangle and DE is drawn parallel to BC cutting the other sides at D and E. If
AB  3.6cm , AC  2.4cm and AD  2.1cm , then AE equal to -
(A) 1.4 cm (B) 1.8 cm (C) 1.2 cm (D) 1.05 cm

8. In a right angled triangle, one of the angles is 60 . The side opposite to this angle is –
1 1
(A)  hypotenuse (B)  hypotenuse
2 2
2 3
(C)  hypotenuse (D)  hypotenuse
3 2

9. In ABC , AD is the median through A and E is the midpoint of AD and BE produced meets
AC in F. Then, AF is equal to –
1 1
(A) AC (B) AC
5 4
1 1
(C) AC (D) AC
3 2

FEAT Educational Ventures Pvt. Ltd.


H.O. 83, Heera Panna Mall, Near Jal-Vayu Vihar, Hiranandani Garden, Powai, Mumbai - 76 119
Call: +91-22-3226-8201 | SMS: +91-842-482-1664 | www.edventure.in | info@edventure.in
Foundation – IX Mathematics Geometry

10. In the adjoining figure, AB = AC and AP  BC. Then,

(A) AB = AP (B) AB < AP (C) AB > AP (D) AB  AP

11. For a triangle ABC, which of the following is true?


(A) BC2 – AB2 = AC2 (B) AB – AC = BC
(C) (AB – AC) > BC (D) (AB – AC) < BC

12. If ABCD is a square and DCE is an equilateral triangle in the given figure, then
DAE is equal to –
1
(A) 45 (B) 30 (C) 15 (D) 22 
2

13 D, E, F are the mid points of the sides BC, CA and AB respectively of ABC . Then DEF is
congruent to triangle –
(A) ABC (B) AEF (C) BFD, CDE (D) AFE, BFD, CDE

14. If in the  's ABC and DEF, angle A is equal to angle E, both are equal to 40 , AB : ED =
AC: EF and angle F is 65 , then angle B is –
(A) 35 (B) 65 (C) 75 (D) 85

15. In the adjoining figure, if BC = a, AC = b, AB = c and CAB  120 ,


then the correct relation is –
(A) a2 = b2 + c2 + 2 bc (B) a2 = b2 + c2 – 2 bc
2 2 2
(C) a = b + c + bc (D) a2 = b2 + c2 – bc

16. O is any point on the bisector of the acute angle XYZ . The line OP is parallel to ZY. Then,
YPO is –

(A) Scalene (B) Isosceles but not right angled


(C) Equilateral (D) Right d & isosceles

17. In a right angled ABC , right angled at A, if AD  BC such that AD = p, If BC = a, CA = b


and AB = c, then –
1 1 1
(A) p2 = b2 + c2 (B) 2
 2  2
p b c
p p
(C)  (D) p2 = b2 c2
a b

FEAT Educational Ventures Pvt. Ltd.


H.O. 83, Heera Panna Mall, Near Jal-Vayu Vihar, Hiranandani Garden, Powai, Mumbai - 76 120
Call: +91-22-3226-8201 | SMS: +91-842-482-1664 | www.edventure.in | info@edventure.in
Foundation – IX Mathematics Geometry

18 In a right angled triangle ABC in which A  90 , If AD  BC, then


the correct statement is-
(A) AB2 = BD  DC (B) AB2 = BD  AD
(C) AB2 = BC  DC (D) AB2 = BC  BD

19. In the adjoining figure, XY is parallel to AC. If xy divides the triangle into equal parts, then the
AX
value of is equal to –
AB
1 1
(A) (B)
2 2
2 1 2 1
(C) (D)
2 2

20. The ratio of the corresponding sides of two similar triangles is 1 : 3. The ratio of their
corresponding heights is –
(A) 1 : 3 (B) 3 : 1 (C) 1 : 9 (D) 9 : 1

21. The areas of two similar triangles are 49 cm2 and 64 cm2 respectively. The ratio of their
corresponding sides is–
(A) 49 : 64 (B) 7 : 8 (C) 64 : 49 (D) None of these

22. The areas of two similar triangles are 12 cm2 and 48 cm2. If the height of the smaller one is
2.1 cm, then the corresponding height of the bigger one is –
(A) 4.41 cm (B) 8.4 cm (C) 4.2 cm (D) 0.525 cm

23. In the adjoining figure, ABC and DBC are two triangles on the same base BC, AL  BC and
area  ABC 
DM  BC. Then, is equal to –
area  DBC 
AO AO2
(A) (B)
OD OD2
AO OD2
(C) (D)
OD AO2

24. In the adjoining figure, AB = 10 cm, BC = 15 cm AD : DC = 2 : 3,


then ABC is equal to –
(A) 30 (B) 40
(C) 45 (D) 110

25. In ABC , D and E are points on AB and AC respectively such that DE || BC. If AE = 2 cm,
EC = 3 cm and BC = 10 cm, then DE is equal to –
20
(A) 5 cm (B) 4cm (C) 15 cm (D) cm
3

FEAT Educational Ventures Pvt. Ltd.


H.O. 83, Heera Panna Mall, Near Jal-Vayu Vihar, Hiranandani Garden, Powai, Mumbai - 76 121
Call: +91-22-3226-8201 | SMS: +91-842-482-1664 | www.edventure.in | info@edventure.in
Foundation – IX Mathematics Geometry

26. In ABC , the medians BE and CF intersect at G. AGD is a line meeting BC in D. If GD is


1.5 cm, then AD is equal to –

(A) 2.5 cm (B) 3 cm (C) 4 cm (D) 4.5 cm

27 In the given figure, ABC  90 and BM is a median, AB = 8 cm and BC = 6 cm. Then,
length BM is equal to –

(A) 3 cm (B) 4 cm (C) 5 cm (D) 7 cm

28. In an equilateral triangle PQR, if p, q and r denote the lengths perpendiculars from P, Q, R
respectively on the opposite sides, then –
(A) p  q  r (B) p = q = r (C) p  q  r (D) p  q  r

29. In the given figure, AE and BD are two medians of a ABC meeting at F. The ratio of the
area of ABF and the quad. FDCE is –

(A) 1 : 1 (B) 2 : 1 (C) 1 : 2 (D) 2 : 3

30 ABC is a triangle and O is the point of intersection of its medians then –


(A) 3 (AB2 + BC2 + CA2) = 4 (OA2 + OB2 + OC2)
(B) 4 (AB2 + BC2 + CA2) = 3 (OA2 + OB2 + OC2)
(C) AB2  BC2  CA2  3 OA2  OB2  OC2
(D) AB2 + BC2 + CA2 = 3 (OA2 + OB2 + OC2)

31. If D, E, F are respectively the mid points of the sides BC, CA and AB of ABC and the area
of ABC is 24 sq. cm, then the area of DEF is –
(A) 24 cm2 (B) 12 cm2 (C) 8 cm2 (D) 6 cm2

32. If AB, m A and m C are given, the number of triangles that can be constructed with this
data is –
(A) One (B) Two (C) Three (D) Nil

FEAT Educational Ventures Pvt. Ltd.


H.O. 83, Heera Panna Mall, Near Jal-Vayu Vihar, Hiranandani Garden, Powai, Mumbai - 76 122
Call: +91-22-3226-8201 | SMS: +91-842-482-1664 | www.edventure.in | info@edventure.in
Foundation – IX Mathematics Geometry

33. If O is a point inside a triangle ABC, which of the following is true?


(A) (AO + BO + CO) < (AB + BC + CA) (B) AO + BO + CO > (AB + BC + CA)
(C) AO + BO + CO = AB + BC + CA (D) None of these

34. In a right angled triangle, if the square of the hypotenuse is twice the product of the other two
sides, then one of the angles of the triangle is –
(A) 15 (B) 30 (C) 45 (D) 60

35. A 25 m long ladder is placed against a vertical wall inside a room such that the foot of the
ladder is 7 m from the foot of the wall. If the top of the ladder slides 4 m downwards, then the
foot of the ladder will slide by –
(A) 16 m (B) 8 m (C) 4 m (D) 2 m

36. EFG and LMN are congruent to each other if :


(A) EFG  LMN , FEG  MLN and FGE  MNL
(B) EF = LM, EG = LN and EFG  LMN
(C) EF = LM, EFG  LMN and FEG  LNM
(D) FG = MN, EG = LN and EGF  LNM

37. If P is any point in the interior of a ABC and PD, PE and PF are perpendiculars to BC, AC
and AB respectively, then –
(A) AE2 + CD2 + BF2 = AF2 + BD2 + CE2 (B) BP2 + PD2 = BD2
2 2 2 2
(C) AE + CD + AB = AF (D) AE2 + AP2 = EP2

38. The in circle of ABC touches BC, CA and AB at D, E and F respectively. The angles of
DEF are –
(A) 180  A , 180  B , 180  C (B) 90  A , 90  B , 90  C
A B C A B C
(C) 90   , 90   , 90   (D) 90   , 90   , 90  
2 2 2 2 2 2

39. ABC is such that AB = 3 cm, BC = 2 cm and CA = 2.5 cm. DEF is similar to ABC .
If EF = 4 cm, then the perimeter of DEF is –
(A) 7.5 cm (B) 15 cm (C) 22.5 cm (D) 30 cm

40. In a ABC , a line XY parallel to BC cuts AB at X and AC at Y. If BY bisects XYC , then –


(A) BC = CY (B) BC = BY (C) BC  CY (D) BC  BY

41. Which of the following is true in the given figure, where AD is the
altitude to the hypotenuse of a right angled ABC ?
1. ABD and CAD are similar
2. ADB and CDA are congruent
3. ADB and CAB are similar
Select the correct answer using the codes given below:
(A) 1 and 2 (B) 1 and 3 (C) 2 and 3 (D) 1, 2 and 3

FEAT Educational Ventures Pvt. Ltd.


H.O. 83, Heera Panna Mall, Near Jal-Vayu Vihar, Hiranandani Garden, Powai, Mumbai - 76 123
Call: +91-22-3226-8201 | SMS: +91-842-482-1664 | www.edventure.in | info@edventure.in
Foundation – IX Mathematics Geometry

42. In the given figure, DE || BC. If AD : DB = 3 : 1 and EA = 3.3 cm, then the length of AC will
be

(A) 1.1 cm (B) 4 cm (C) 4.4 cm (D) 5.5 cm

43. XY is drawn parallel to the base BC of a ABC cutting AB at X and AC at Y. If AB = 4 BX


and YC = 2 cm, then AY will be –
(A) 2 cm (B) 4 cm (C) 6 cm (D) 8 cm

AB BC CA 2
44. If in ABC and DEF ,    , then their areas are in the ratio –
DE EF DB 5
(A) 2 : 5 (B) 4 : 25 (C) 4 : 15 (D) 8 : 125

45 Let ABC be an equilateral triangle. Let BE  CA meeting CA at E. Then, (AB2 + BC2 + CA2)
is equal to –
(A) 2 BE2 (B) 3 BE2 (C) 4 BE2 (D) 6 BE2

BD
46. ABC is a right angled triangle at A and AD is perpendicular to the hypotenuse. Then, is
DC
equal to –
2 2
 AB  AB  AB  AB
(A)   (B) (C)   (D)
 AC  AC  AD  AD

47. In a ABC , perpendicular AD from A to BC meets BC at D. If BD = 8 cm, DC = 2 cm and


AD = 4 cm, then –
(A) ABC is an isosceles triangle (B) ABC is an equilateral triangle
(C) AC = 2 AB (D) ABC is right angled at A

48. If ABC and DEF are similar, 2AB = DE and BC = 8 cms. Then, EF is equal to –
(A) 16 cms (B) 12 cms (C) 8 cms (D) 4 cms

49. If ABC is an isosceles triangle and perpendicular AD is drawn from the vertex A to any point
D on the base, then –
(A) AB2 – AD2 = BD . DC (B) AB2 + AD2 = BD . DC
(C) AB2 – AD2 = BD2 – DC2 (D) AB2 + AD2 = BD2 – DC2

50. In the given figure, ABC is a non-isosceles right angled triangle, B being
a right angle. EF and BG are perpendiculars on the hypotenuse AC.
Then, CEF is equal to –
(A) ABG (B) BAG
(C) ECF (D) EFC

FEAT Educational Ventures Pvt. Ltd.


H.O. 83, Heera Panna Mall, Near Jal-Vayu Vihar, Hiranandani Garden, Powai, Mumbai - 76 124
Call: +91-22-3226-8201 | SMS: +91-842-482-1664 | www.edventure.in | info@edventure.in
Foundation – IX Mathematics Geometry

51. If in triangles ABC and DEF, A  E  40 , AB : ED = AC : EF and F  65 , then B is–
(A) 35 (B) 65 (C) 75 (D) 85

52. If ABC and DEF are similar triangles in which A  47 and E  83 , then C is –
(A) 50 (B) 60 (C) 70 (D) 80

53. In triangles ABC and PQR, m  B   m  Q  , AB : AC = PR : QR. If m  A   m  P  and,


m  R   73 then m  C  will be –
(A) 17 (B) 73 (C) 117 (D) 107

54. If ABC is a triangle right angled at B and M, N are the mid points of AB & BC, then
 
4 AN2  CM2 is equal to –
5
(A) 4AC2 (B) 5AC2 (C) AC2 (D) 6AC2
4

55. If ABC is an equilateral triangle and AD  BC, then AD2 is equal to –


3
(A) DC2 (B) 2DC2 (C) 3DC2 (D) 4DC2
2

AB BC CA
56. If ABC and DEF are so related that   , then which of the following is true?
FD DE EF
(A) A  F and B  D (B) C  F and A  D
(C) B  F and C  D (D) A  E and B  D

57. In the adjoining figure, AB || PQ, AC || PR, BC || QR. Then, AP is –


(A) A median of PQR
(B) The angular bisector of QPR
(C) Perpendicular to QR
(D) None of these

58. In a right angled ABC , right angled at B, if P and Q are points on the sides AB and AC
respectively, then –
(A) AQ2 + CP2 = 2 (AC2 + PQ2)
(B) 2 (AQ2 + CP2) = (AC2 + PQ2)
(C) AQ2 + CP2 = AC2 + PQ2
1
(D) AQ + CP = (AC + PQ)
2

59. The areas of two similar triangles are 81 cm2 and 144 cm2. If the largest side of the smaller
triangle is 27 cm, then the largest side of the larger triangle is –
(A) 24 cm (B) 36 cm (C) 48 cm (D) None of these

60. In a ABC , A  90 , AB = 5 cm and AC = 12 cm. If AD  BC, then length of AD is –


13 2 15 60 13
(A) cm (B) cm (C) cm (D) cm
2 13 13 60

FEAT Educational Ventures Pvt. Ltd.


H.O. 83, Heera Panna Mall, Near Jal-Vayu Vihar, Hiranandani Garden, Powai, Mumbai - 76 125
Call: +91-22-3226-8201 | SMS: +91-842-482-1664 | www.edventure.in | info@edventure.in
Foundation – IX Mathematics Geometry

61. D and E are the points on the sides AB and AC respectively of a ABC . In which of the
following cases DE || BC?
(A) AD = 5 cm, BD = 6 cm, AE = 6 cm, CE = 5 cm
(B) AB = 18 cm, AD = 8 cm, AE = 12 cm, EC = 15 cm
(C) AD = 3 cm, BD = 8 cm, AC = 8 cm, AE = 3 cm
(D) None of these

62. In the adjoining figure, the exterior bisector of BAC meets BC produced at D. If
AB  6 cm , BC  4 cm and AC  5 cm then CD is equal to –

(A) 24 cm (B) 30 cm (C) 20 cm (D) 8.3cm

LM ND
63. If D is such a point on the side MN of LMN that  , then LD must be a/an –
LN DN
(A) Median of LMN (B) Bisector of MLN
(C) Altitude of LMN (D) None of these

64. In the adjoining figure, ADE  B , AE = 8 cm, EB = 7 cm, BC = 9 cm,


AD = 10 cm and DC = 2 cm. Then the length DE is –
(A) 6 cm (B) 13.5 cm
(C) 6.75 cm (D) 7.8 cm

65. In ABC shown in the figure, m A  90 . Let D be a point on BC such


that BD : DC = 1 : 3. If DM and DL respectively are perpendiculars on AB
and AC, then DM and LC are in the ratio of
(A) 1 : 4 (B) 2 : 3
(C) 1 : 3 (D) 1 : 2

66. The alternate sides of a hexagon are produced to meet so as to from a star
shaped figure, as shown. The sum of the angles at the vertices of the star is
(A) 2 right angles (B) 8 right angles
(C) 4 right angles (D) 6 right angles

FEAT Educational Ventures Pvt. Ltd.


H.O. 83, Heera Panna Mall, Near Jal-Vayu Vihar, Hiranandani Garden, Powai, Mumbai - 76 126
Call: +91-22-3226-8201 | SMS: +91-842-482-1664 | www.edventure.in | info@edventure.in
Foundation – IX Mathematics Geometry

EXERCISE – IV
1. PQRS is a parallelogram and line-segments PA and RB bisect the angles P and R
respectively. Show that PA II RB.

2. In the figure  ABC, lines are drawn parallel through A, B and C respectively to the sides
1
BC, CA and AB forming  PQR. Show that BC  QR .
2

3. Three angles of a quadrilateral are equal and the measure of the fourth angle is 165o . Then
the measure of each of the equal angles is
(A) 45 o (B) 55 o (C) 65 o (D) 75 o

4. The bisectors of  A and  B meet at a point P. If  C= 100o and  D = 50 o then the


measure of  APB is equal to __________.
(A) 45 o (B) 55 o (C) 65 o (D) 75 o

5. To draw a quadrilateral, the number of independent measures required is __________.

6. A square of side ‘a’ units is combined with two right-angled triangles of base ‘  ’ units and ‘y’
units respectively to form a trapezium, as shown in the figure. Find the sides of the
trapezium formed. [Hint: Use Pythagoras theorem i.e., if a, b, c are three sides of a right-
angled triangle with ‘C’ as hypotenuse then a2  b2  c 2 ]

7. A polygon has 9 sides. Into how many triangles can the polygon be partitioned by the
diagonals drawn from one vertex?

8. The exterior angle of a polygon, whose angles are all equal, is 72°. How many sides does
the polygon possess?

9. The interior angle of a regular polygon is three times the exterior angle. Find the number of
sides in the polygon.

10. The sum of the measures of the interior angles of a regular polygon is twice the sum of the
measures of tis exterior angles. Find the number of sides of the polygon.

11. An eight-sided polygon has four angles each of measure 1540 . Calculate the measure of
each of the remaining angles, which are all equal.

FEAT Educational Ventures Pvt. Ltd.


H.O. 83, Heera Panna Mall, Near Jal-Vayu Vihar, Hiranandani Garden, Powai, Mumbai - 76 127
Call: +91-22-3226-8201 | SMS: +91-842-482-1664 | www.edventure.in | info@edventure.in
Foundation – IX Mathematics Geometry

12. Prove that the sum of all the exterior angles formed by producing the sides of a convex
pentagon in the same order is equal to four right angles.

13. The number of diagonals in a octagon is __________.

14. A regular polygon is inscribed in a circle. If a side subtends an angle of 72o at the centre,
what are the numbers of sides of the polygon?

15. (I) what is the sum of the angles of a pentagon?


(II) The sum of the interior angles of a polygon is 1260 . How many sides does the polygon
possess?

1
16. S is the sum of the interior angles of a polygon P, for which each interior angle is 7 times
2
the exterior angle at the same vertex. Then if
(I) S  3060o , P may be regular
(II) S  3060o , P is not regular
(III) S  2700o , and P is regular
(IV) S  2700o , and P may or may not be regular.

17. One regular polygon P1 has n sides; another regular polygon P2 has 2n sides. What is the
ratio of the exterior angle of P1 to the exterior angle of P2 ? Will the same ratio hold well in
respect of the interior angle?

FEAT Educational Ventures Pvt. Ltd.


H.O. 83, Heera Panna Mall, Near Jal-Vayu Vihar, Hiranandani Garden, Powai, Mumbai - 76 128
Call: +91-22-3226-8201 | SMS: +91-842-482-1664 | www.edventure.in | info@edventure.in
Foundation – IX Mathematics Geometry

ANSWER KEY

EXERCISE – I

1. (30º, 60º, 90º) 7. (50º, 60º, 70º)

8. (360º) 9. (130º)

14. (i) 20 cm; (ii) 6.43 cm; (iii) 7.2 cm; (iv) 11 cm; (v) 6 cm; (vi) 17 cm; (vii) 3 cm; (viii) 2 cm;
(ix) x = 4; (x) x = 1; (xi) x = 1

16. (i) 2.1 cm; (ii) 7.5 cm; (iii) 2.3 cm; (iv) 2.5 cm, 3.5 cm; (v) 2.8 cm; (vi) 5.8 cm; (vii) 4.9 cm;
(viii) 7.5 cm, 4.5 cm

26. (5º) 27. (17.5º)

28. (120º) 39. (110º)

43. (85º, 60º)

EXERCISE – II

1. (12 : 15 : 20) 2. (50º, 115°)

3. (A) 4. (45º)

5. (B) 6. ADC )

7. (A) 8. (100º)

12. (MN) 15. (20°)

17. (18) 18. (5/3 cm; 32/13 cm; 40/9 cm)

19. ((I) 3; (II) 2; (III) 11 or 8)

EXERCISE – III

1. (D) 2. (C) 3. (C) 4. (B) 5. (C) 6. (C) 7. (A)

8. (D) 9. (C) 10. (C) 11. (D) 12. (C) 13 (D) 14. (C)

15. (C) 16. (B) 17. (B) 18 (D) 19 (B) 20. (A) 21. (B)

22. (C) 23. (A) 24. (B) 25. (B) 26. (D) 27 (C) 28. (B)

29. (A) 30 (D) 31. (D) 32. (B) 33. (D) 34. (C) 35. (B)

FEAT Educational Ventures Pvt. Ltd.


H.O. 83, Heera Panna Mall, Near Jal-Vayu Vihar, Hiranandani Garden, Powai, Mumbai - 76 129
Call: +91-22-3226-8201 | SMS: +91-842-482-1664 | www.edventure.in | info@edventure.in
Foundation – IX Mathematics Geometry

36. (D) 37. (A) 38. (A) 39. (B) 40. (A) 41. (D) 42. (C)

43. (C) 44. (B) 45 (C) 46. (B) 47. (D) 48. (A) 49. (A)

50. (B) 51. (C) 52. (A) 53. (B) 54. (B) 55. (C) 56. (A)

57. (A) 58. (C) 59. (B) 60. (C) 61. (C) 62. (C) 63. (B)

64. (A) 65. (C) 66. (C)

EXERCISE – IV

3. (C) 4. (D)

5. (Five measures are required) 7. (7)

8. (n = 5) 9. (8)

10. (n = 6) 11. ( 116 )

13. (20) 14. (n = 5)

15. (i) 540 ; (ii) n = 9 17. ( 2 : 1)

FEAT Educational Ventures Pvt. Ltd.


H.O. 83, Heera Panna Mall, Near Jal-Vayu Vihar, Hiranandani Garden, Powai, Mumbai - 76 130
Call: +91-22-3226-8201 | SMS: +91-842-482-1664 | www.edventure.in | info@edventure.in
About Us
Edventure is a well-crafted institution with a team of dedicated experienced IITians who intend to help an
individual succeed in today's tough competition. In the times of continuously degrading education quality, we
are continuously trying to establish a strong and quality platform for engineering and medical entrance
preparations.We are continuously working to provide the best in the field.

IITs/AIIMs are the most prestigious institutes of our country. Getting into IITs/AIIMs has been the toughest
challenge for every student. In this competitive world, every student has to deal with the pressure of
performing well. We understand each child has a different set of skills and talent which needs to be nurtured.
The learning phase of one's life is a constantly evolving phase which is very critical in molding the future.

Key features of Edventure:


· Personalised Coaching for engineering and Medical students
· Small Batch size having only 12 student in a batch
· Experienced IITians / Ex-IITians PACE faculty team
· Limited Intake of students insuring personalised touch with quality education

Edventure FEAT Educational Ventures Pvt. Ltd.


H.O. : 83, Heera Panna Mall, Hiranandani, Powai, Mumbai - 400076

Bandra
305, Rizvi Chamber,
Near Lucky Restaurant,
Hill Road, Bandra (W)

www.edventure.in | info@edventure.in | +91-22-3226-8201

You might also like